B/B Exam Q'S

Pataasin ang iyong marka sa homework at exams ngayon gamit ang Quizwiz!

Which of the following are products of the pentose phosphate pathway? I. NADPH II. Glycolytic intermediates III. Ribose-5-phosphate A) I only B) II only C) I and III only D) I, II, and III

D

Each of the following structures are derived from the mesoderm EXCEPT: A) Kidneys. B) Spinal cord. C) Triceps. D) Circulatory system.

B - The mesoderm gives rise to the musculo-skeletal system, circulatory system, and kidneys. The spinal cord is derived from ectoderm. A, C, D: This structure develops from the mesoderm.

Spina bfida, or cleft spine, is a birth defect caused by incomplete neurulation and resulting in nerve damage. This condition is the produce of the improper development of the: A) Ectodermal germ layer B) Endodermal germ layer C) Medodermal germ layer D) Diploblasmal germ layer

A -Nervous system is derived from the ectodermal; b- the endoderm is the most internal layer forms the linings of many organs c- mesoderm is middle; blood, muscle and kidney cells d- not a germ layer

Phosphate group

A phosphate group is both polar and negatively charged, so it would not be able to pass through the hydrophobic cellular membrane. One of the benefits of phosphorylation is that it restrains the molecule and keeps it from exiting the cell.

APE Sites

A site receives incoming tRNAs, P site is the region of the ribosome in which peptide tRNAs are formed; the tRNA molecule actually forms a complex with the growing polypeptide at this site; additional residues are added via the formation of peptide bonds, E site serves as the location where the newly formed polypeptide chain exit

All of the following are functions of mammalian skin EXCEPT: A) Sensation. B) Respiration. C) Protection from disease. D) Protection against internal injury.

B

The Arf1-activating molecule GTP is most closely related to which family of biomolecules? A) Nucleotides B) Amino acids C) Lipids D) Carbohydrate

A: GTP stands for guanosine tri-phosphate, which is a nucleotide.

Enzymes

An enzyme reduces the activation energy for both the forward and reverse reactions. The direction of reaction in this case is governed by the relative amounts of reactants and products, i.e. the difference between the reaction quotient and equilibrium. In tissue, high amounts of carbon dioxide exist outside of the red blood cell, driving the fusion into the sale and consumption of the carbon dioxide by carbonic anhydrase. In the lungs, there is low carbon dioxide tension outside of the cell, driving production of the gas by carbonic anhydrase and subsequent diffusion out of the cell into the blood.

A fatty acid is comprised of a long hydrocarbon tail and a head consisting of: A) A hydroxyl group. B) A carboxyl group. C) A phosphate group. D) An amino group.

B

The structure of a typical nucleoside includes: A) A six-carbon sugar B) A nitrogenous base C) Phosphate Groups D) A and B

B - A nucleoside contains both a sugar and a nitrogenous base, but unlike a nucleotide, it does not include phosphate groups. The sugar involved is either a ribose or deoxyribose, both of which have five carbons.

Patients with central diabetes insipidus (CDI) excrete large volumes of urine with low osmolarity. Given this, the nephrons of CDI patients most likely possess: A) Loops of Henle that penetrate into the inner region of the medulla. B) Collecting ducts composed of epithelial cells deficient in functional aquaporins C) Large numbers of Na+/K+-ATPases in the cells of the proximal tubules. D) Highly active urea transporters in the distal portions of the collecting ducts

B - A primary function of the collecting duct is reabsorption of water. This process is regulated by antidiuretic hormone, which increases the insertion of aquaporins into the plasma membranes of collecting duct epithelial cells. As a result, collecting ducts become more permeable to water, increasing reabsorption of water from the filtrate and decreasing the volume (ie, water content) of excreted urine. Decreasing the water volume of urine also increases its solute concentration (ie, osmolarity). This question states that patients with central diabetes insipidus (CDI) produce large volumes of urine with low osmolarity (ie, dilute urine). Because collecting duct aquaporins reabsorb water into the body, their production of excess dilute urine is likely due to collecting ducts that are deficient in functional aquaporins. The other options would increase water reabsorption, decreasing (not increasing) urine volume: (Choice A) Countercurrent multiplication in the loop of Henle contributes to the formation of a concentration gradient such that water reabsorption from filtrate is maximized. Therefore, nephrons with long loops of Henle penetrating into the inner region of the medulla (ie, juxtamedullary nephrons) would increase water reabsorption. (Choice C) Proximal tubule Na+/K+-ATPases (ie, sodium-potassium pumps) actively transport sodium ions from the filtrate to the extracellular region outside the tubule. This allows more water to be reabsorbed as it flows via osmosis from the less concentrated filtrate in the tubule to areas of higher solute concentration outside the tubule. (Choice D) Urea transporters in the distal collecting duct reabsorb urea (solute) from the filtrate into the medulla. This increases medullary solute concentration and therefore increases the reabsorption (osmosis) of water from the filtrate into the salty medulla.

Metabolism of which fuel use the largest ATP output under aerobic conditions? A) Sugars. B) Fatty acids. C) Proteins. D) Vitamins.

B - Fatty acids are the most energy dense fuel taken in by the body. The longer the fatty acid chain, the more ATP can be produced in metabolism. Proteins can be used in metabolism when there is a lack of other fuel sources available, fats and sugars. Catabolism of proteins yields more ATP than sugars but less than fats.

Which of the following are found in both prokaryotic and eukaryotic mechanisms of transcriptional regulation? I. Promoter sequences II. Operons III. Enhancers IV. Repressors A) I only B) I and IB C) I, II, and III D) I, III, and IV

B - the promoter is the region of DNA to which rna polymerase binds to initiate transcription; essential in both Repressors are proteins that inhibit gene expression by blocking specific regions of DNA or by preventing rna polymerase from binding to the promoter; found in both ii. operons are functional groups of genes controlled by a single promoter; only in prokaryotes iii, enhancers are regions of DNA to which proteins can bind and activate transcription; only in eukaryotes

Which of the following is NOT a function of the sympathetic nervous system? A) Increased heart rate B) Pupillary constriction C) Vasodilation D) Vasoconstriction

B- Activation of the sympathetic nervous system triggers a fight-or-flight response. This includes several physiological responses: pupil dilation, increased rate and force of heart contraction, blood vessel dilation in skeletal muscle and constriction in gastrointestinal organs, and inhibition of peristalsis by the digestive tract. The parasympathetic system has the opposite effect. The SNS causes vasodilation in skeletal muscle to ready the body for "fight or flight" and promotes vasoconstriction to the viscera, slowing down the digestive tract.

Which of the following is an element of humoral immunity? A) Phagocytes B) Immunoglobulins C) T cells D) MHC I

B- Humoral immunity is part of the body's adaptive immune response. It is provided by B cell activity, which promotes an antibody, or immunoglobulin, response. Antibodies can recognize polysaccharide, phospholipid, and nucleic acid antigens to help the body fight extracellular bacteria, viruses, and toxins. While phagocytes, T-cells and MHC I are all part of cell-mediated immunity

A human cell has only 13 chromosomes. What type of cell is it? A) A muscle cell B) A germ cell C) A diploid cell D) A somatic cell

B- This cell has an odd number of chromosomes, meaning that it must be a haploid cell. Only germ cells are haploid.

GAP belongs to what class of enzymes? A) Transferase B) Phosphatase C) Kinase D) Isomerase

B: The passage notes that GAP catalyzes the conversion of GTP to GDP and inorganic phosphate, inferring that it is a phosphatase. Phosphatases are responsible for the cleavage of phosphate bonds utilizing water to remove a molecule of inorganic phosphate.

BME (beta mercaptoethanol)

BME denatures proteins tertiary structure.

RNA vs DNA stability

Because RNA contains ribose, the presence of a 2' OH group makes RNA more reactive and unstable than DNA

PCR Primers

Best primers for PCR have a high GC content and CG bases in 5′ and 3′.

How does the initiation step of transcription differ from that of DNA replication? A) Replication involves a DNA dependent DNA polymerase, while transcription utilizes an RNA dependent RNA polymerase B) DNA replication is semiconversative and uses Okazaki fragments, while transcription forms a single stranded product C) Transcription relies on sigma factors while DNA replication uses particular points in the DNA known as origins D) The process do not differ notable; bother require RNA primers due to the free OH groups at their 3' ends

C- Transcription uses a DNA dependent RNA polymerase; requires a DNA transcript but catalyzes the lengthening of an RNA strand b) this describes the elongation step of replication and the final product of transcription

Translation is the process by which protein is produced from mRNA. From beginning to end, the four phases are: A) Translocation, initiation, elongation, and termination B) Initiation, translocation, elongation, and termination C) Translocation, elongation, initiation, and termination D) Initiation, elongation, translocation and termination

D

During the Krebs cycle, isocitrate and NAD come together to form alpha ketoglutarate, and ADH and H. Which of the following describes this type of chemical reaction? A) Dehydration. B) Hydration. C) Decarboxylation. D) Oxidation.

D - When alpha ketoglutarate is formed, isocitrate loses electrons. The loss of electrons characterizes an oxidation reaction. During this reaction, and a D is simultaneously reduced form NADH and H. Dehydration reactions result in the loss of water, typically to a leaving group or in this creation of a double bond. Hydration reactions involve the addition of water. A decarboxylation reaction results in the loss of carbon dioxide.

Kd/Km

Furthermore, the lower the Kd of the drug (0.25 mM), the more effective it will b

Zymogens

Hydrolysis mechanisms or when a molecule of water is used to split the substrate. Hydrolysis is a very common method of activating zymogens.

AMPA receptors

Increased conduction through AMPA receptors would permit the more efficient passage of extracellular cations into th neuron, facilitating a more rapid postsynaptic response. The activation of scaffolding proteins would permit tight co localization of AMPA receptors on the postsynaptic membrane effectively increasing the likelihood of glutamate binding. Scaffolding proteins prevent diffusion of transmembrane AMPA receptors out of he postsynaptic density, which allows the postsynaptic neuron to remain sensitive to presynaptic inputs.

Phosphates

Inorganic phosphate is incorporated onto the three carbon Whiteys in the second half of glycolysis through coupling of this reaction with the reaction of a NAD. THE ADDITION OF A PHOSPHATE IS COUPLED WITH A REDUCTION OF NAD TO NADH.

KM

KM is the substrate concentration at which the reaction rate is one-half the maximal velocity, not at one-half the substrate concentration.

Lactate

Lactate itself cannot be metabolized further into producing any more ATP. However, it can be reincorporated into a glucose molecule through gluconeogenesis. While glycolysis only produces two ATP per glucose molecule, the reaction rate is very rapid and that is sufficient power oxygen deprived muscles.

Negative control

Negative control involves the binding of a repressor to block transcription, while positive control includes an activator protein that binds and stimulates transcription; if an operon is switched on by an inducer, indicating an induction system Positive operon control involves an activator protein that binds and stimulates transcription

DNA Synthesis

Nucleotides can combine to form long chains known as polynucleotides or nucleic acids (ie, RNA or DNA), as catalyzed by RNA polymerase or DNA polymerase. In biological systems, lengthening of these chains occurs from the 5′ end to the 3′ end (ie, new nucleotides are added to the 3′ end of the chain). During this process, the 3′ hydroxyl group of the growing chain acts as a nucleophile to attack the 5′α-phosphate of the incoming nucleotide.

L amino acids

Only L amino acids are produced in cells and used to form protein. Although de amino acids are occasionally incorporated into bacterial cell walls, they are not used in protein production.

What are snRNPs comprised of?

RNA and protein; stands for small nuclear ribonucleic protein

Sickle cell anemia

Sickle cell anemia is exacerbated by increased concentrations of deoxyhemoglobin. This form of hemoglobin predominates under conditions of hypoxia, low pH, and high carbon dioxide. Vigorous exercise and metabolic acidosis will promote the conversion of oxyhemoglobin to deoxyhemoglobin. III. THIS WILL CAUSE MORE MOLECULES TO ASSUME THE OXYHEMOGLOBIN FORM, WHICH HAS A LOWER AFFINITY FOR OTHER HEMOGLOBIN MOLECULES.

Rh Antigen

The mother will only mount an immune response to an antigen that her immune system recognizes as foreign. For this region, she must be negative for the Rh antigen. The formation of antibodies and the generation of a memory humoral response take between 48 and 96 hours. This means the first pregnancy is at minimal risk for hemolytic disease of the newborn. However, the maternal immune system has been exposed to fetal Rh. Thus subsequent exposure to Rh will elicit a much stronger response.

Parasympathetic System

The peripheral nervous system is divided into: the sensory, or afferent, division, which is involved in transmitting impulses to the central nervous system (brain and spinal cord), and the motor, or efferent, division, which is involved in sending impulses to peripheral tissues and systems. The motor division is subdivided into two branches: the somatic nervous system, which is responsible for activating voluntary skeletal muscle. the autonomic nervous system, which activates involuntary smooth muscle, cardiac muscle, and responses by glands (eg, adrenal gland). Finally, the autonomic nervous system is subdivided into two categories: The sympathetic ("fight or flight") division is responsible for the mobilization of energy in stressful situations (ie, immediate danger). For example, stimulation of sympathetic division nerve fibers activates processes that lead to effects like increased heart rate and dilation of airways. At the same time, sympathetic activation inhibits activities such as digestion so that energy can be redirected toward addressing stressors. The parasympathetic ("rest and digest") division is responsible for energy storage when stress is low. For example, stimulation of the parasympathetic division promotes digestive activity and normal urination patterns.

Reaction quotient

The reaction quotient is measured as a relative concentrations of the products divided by that of the reactance. Due to large amounts of carbon dioxide produced by respiration during exercise, there is a larger than usual concentration of CO2, a reactant, present. This makes Q in unusually small number. When Q is less than the equilibrium value K, the reaction favors a product. This means that the system shifts to the right to make the products in order to reestablish equilibrium.

Size Exclusion Chromatography

The use of one solvent or another shouldn't make a large difference in SEC retention times, since it's size, not polarity, that causes separation (eliminate choices A and B). Long retention times on the SEC column mean that the oligomers are small, which also means they are more volatile and have shorter retention times on GC. The converse is also true: short SEC retention times mean the oligomers are large and less volatile, with longer GC retention times, so choice C is false.

snRNAs

Their major role involves the formation of the spliceosome. This cellular machinery is responsible for removing introns from pre-mRNA transcripts during post-transcriptional processing. If an organism stopped producing snRNAs, then this modification process would be impaired

Sensory afferent tracts

They are located toward the rear, or dorsal side of the spinal cord. In contrast, motor efferent tracts lie toward the front, or ventral, and lateral sides. If only the dorsal nerves are severed ,then motor neurons are still intact the digestive tract is controlled by visceral motor neurons

cDNA

cDNA is made from mRNA that has undergone post-transcriptional modification. These changes eliminate promoter and other regulatory sequences as well as in trans, all of which are still present in the DNA from the genomic library. Post- transcriptional modified DNA is used as a template to make cDNA, not the other way around.

John runs a solution of protein a, B, and C through a size exclusion chromatography column. The proteins dilute out in the following order, from earliest to latest; C, A, B. What can Jon conclude? I. Proteins see you in migrate a shorter distance and an SDS page gel then protein B. II. Protein B has quaternary structure. III three proteins see is comprised of many hydrophobic residues. A) I only B) II only C) III only D) I, II, and III

A - In size exclusion chromatography, proteins that are larger elute faster, as they are less likely to become trapped in the spaces within the stationary phase. In contrast, larger proteins migrate more slowly through a gel electrophoresis. Size exclusion chromatography yields no information on the presence of subunits or the nature of the proteins amino acid composition.

Most bacterial cells and human cells are alike in: A) The ability to produce ATP via ATP synthase. B) The chemical composition of their ribosomes. C) Their enclosure within cell walls. D) The shape of the self-replicating structures that carry their DNA.

A - Of the options listed, only A, the ability to produce ATP via ATP synthase is common to both bacterial and human cells. Both types of cells possess a membrane-embedded electron transport chain capable of generating a H+ gradient, which drives synthesis of ATP via ATP synthase. This ATP synthesis takes place on the plasma membrane of bacteria and on the inner mitochondrial membrane in human cells. The chemical composition of prokaryotic and eukaryotic ribosomes (B), although similar, is distinct enough that several types of antibiotics are able to preferentially target prokaryotic ribosomes over eukaryotic ribosomes. Of human and bacterial cells, only bacterial cells have cell walls (C), and most bacterial chromosomes are circular whereas human chromosomes are linear (D). Thus, A is the best answer.

A physiology teacher is describing the nephron to her class. She mentions that, and healthy individuals, no glucose should be excreted in the urine. Instead, glucose monomers are reabsorbed from the lumen of the nephron into the interstitium surrounding the loop of Henley for later returned to the bloodstream. What mistake, if any, did the teacher make? A) Glucose is removed from the filtrate in the proximal convoluted tubule, not the loop of Henle. B) Healthy humans tend to excrete a significant amount of glucose in the urine. C) The movement of solute out of the lumen of the nephron for retention in the body is termed secretion, not reabsorption. D) None of the above. The teachers explanation is accurate.

A - Overall, the teacher did a somewhat decent job explaining his vital physiological process. However, she incorrectly stated that glucose is reabsorbed in the loop of Henle, a structure that mainly absorbs water and sodium. Most reabsorption of biologically important solutes, including glucose, occurs in the PCT. B: This statement is false. In fact, glucose in one's urine is a classic sign of untreated diabetes. Secretion, or the transport of solutes from the bloodstream into the nephron, is the opposite of the process of the teacher is describing.

A sample of noncoding RNA is isolated from a cell's nucleus, where it was observed to exist in close proximity to pre-mRNA. This sample most likely contains: A) snRNA B) rRNA C) tRNA D) snoRNA

A - Small nuclear RNA molecules are involved in the splicing of pre-mRNA. b. rRNA is produced in the nucleolus and is located within or near ribosomes c. tRNAs are likely to. be found in the cytoplasm d. snoRNAs exist in the nucleolus

During oxidative phosphorylation, which membrane complex transfers electrons from succinate to coenzyme Q? A) Complex I B) Complex II C) Complex III D) Complex IV

B - Complex II, like complex I, transfers electrons to coenzyme Q. Complex I receives its electrons from NADH, while complex II receives electrons from succinate. A: Complex I transfers electrons from NADH to coenzyme Q. C: Complex III transfers electrons from coenzyme Q to cytochrome c. D: Complex IV transfers electrons from cytochrome c to the final electron acceptor, oxygen.

What purpose do primers serve in the initiation of DNA replication? A) They signal the location at which to form the replication fork B) They provide sites to which a vital enzyme can bind C) They indicate which of the two strands of parent DNA is to be replicated D) They disrupt hydrogen bonding between base pairs, allowing the double helix to unwind more easily

B - DNA polymerase catalyzes the elongation of new DNA strands. This enzyme cannot initiate DNA synthesis on its own, as it can only bind to a free OH group that is already present on the developing strand. RNA primers provide these free 3' hydroxyl groups. A) While the replication fork does initially form at the site where replication begins, it then moves along the strands as they are unwound by helicase, the fork does not relate to the direct function of primers. C) Both parent strands are used as templates during DNA replication. D) This more closely relates to the function of helicase

Which sequence of events correctly depicts the conversion of DNA into protein in a eukaryotic cell? A) DNA is translated into mRNA in the nucleus, then transcribed into an amino acid chain in the cytosol B) DNA is transcribed into mRNA in the nucleus, then translated into an amino acid chain in the cytosol C) DNA is transcribed into mRNA in the nucleus, then translated into an amino acid chain in the nucleus D) DNA is transcribed into mRNA in the cytosol, then translated into an amino acid chain

B - DNA resides in the nucleus, the organelle in which is it transcribed into mRNA; Then enters the cytosol, where it is translated into protein at a ribosome

Starch in the digestive system is broken down by which enzyme? A) Lysozyme. B) Amylase. C) Pepsin. D) Trypsin.

B - Two types of amylase exist: salivary and pancreatic. Both of these work to break down starches. A: Lysozyme hydrolyzes B-1,4 linkages in bacterial cell walls. C: Pepsin is a protease released into the stomach that digest protein. D: Trypsin is a pancreatic protease.

Which of the following mutations would most likely lead to a virus incapable of infecting liver cells? A) Missense mutation in pre-S2 B) Nonsense mutation late in S C) Single base pair insertion in pre-S1 D) Three nucleotide deletion early in S

C - A single-base pair insertion would cause a reading frame shift. This would be devastating for the S gene in HBV because it has three start codons, all in-frame. Not only would the start codons be affected, but even if another start codon existed in the new reading frame, the amino acid sequence of the protein would be completely altered. Since the S gene encodes the surface antigen required for entry into liver cells, a virus with this mutation would likely be incapable of infection (choice C is correct). A missense mutation causes the substitution of one amino acid for another. Since pre-S2 is a start codon, this could be bad, but the passage states that the S gene has three in-frame start codons, so it is likely that one of the other two start codons could be used, translation would start, and protein structure would be unaffected (choice A is wrong). A nonsense mutation replaces an amino acid codon with a stop codon. This leads to a truncated protein, but if it is late in the S gene, it may still have some activity (choice C is better than choice B). A three-nucleotide deletion in the S gene would result in the loss of one amino acid (which in some cases could affect the three-dimensional structure of the resulting protein) but this is not as devastating an effect as is a reading frame shift (choice C is better than choice D).

Presumably, hyperglycemia promotes cellular dehydration because: A) Glucose, as an energy source, accelerates the osmotic work performed by plasma membranes. B) Glucose, as an energy source, accelerates plasma membrane ion exchange pumps. C) Glucose molecules raise the osmotic pressure of the extracellular space. D) Glucose molecules are exchanged for water molecules across the plasma membrane.

C - Although glucose is an energy source and can be transported across the plasma membrane, the protection of glucose in this harsh environment is due to its effects on the diffusion of water (i.e., its osmotic effects). Water diffusion does not require ion-exchange pumps. Glucose is not exchanged for water molecules across the membrane. Thus, answer choice C is the best response.

Elevated levels of which substrate will stimulate gluconeogenesis? A) ADP. B) AMP. C) Acetyl Co-A. D) Glucose.

C - High levels of acetyl Co-A indicate high energy production in the cell. Typically all acetyl co-A is shuttled into the citric acid cycle. When levels rise in the cell, however, glycolysis is an activated and gluconeogenesis is stimulated.

Which statement(s) is/are true regarding antigen-presenting cells? I. The only MHC proteins they express are class II. II. They have a direct role in all types of rejection. III. A dendritic cell is one type of APC. A) II only B) I, II, and III C) III only D) I and II only

C - Item I is false: antigen presenting cells (APCs) do express MHC class II proteins, but they also express MHC class I proteins. All nucleated cells express MHC I (choices B and D can be eliminated). Note that determining true or false for only one of the remaining Items will give you the correct answer choice. Item II is false: while APCs play a direct role in acute rejection, the passage states that hyperacute rejection involves pre-existing antibodies, not antigen-presenting cells (choice A is wrong and choice C is correct). Note that Item III is true: dendritic cells are one type of APC

The sequence of a typical HERV is most likely to contain a gene that codes for: A) The production of a DNA-dependent RNA polymerase. B) The production of an RNA-dependent RNA polymerase. C) The production of an RNA-dependent DNA polymerase. D) The production of a helicase enzyme.

C - The passage states that HERVs, or human endogenous retroviruses, arose through the integration of retroviral material into the genome. It also mentions that HERVs are able to perform reverse transcription, a process that requires the enzyme reverse transcriptase. This enzyme catalyzes the production of DNA from an RNA template. Reverse transcriptase thus must have DNA polymerase activity, since it builds a new DNA strand; it is also RNA-dependent, since it reads an RNA template.

Fast which is expressed by most healthy somatic cells, is a membrane receptor that triggers apoptosis when bound by fas ligand. Both fas ligand and fas itself are expressed on the surface of CD8+ T cells, the former of which enables, at least in part, their cytotoxic function. Cells that are least likely to express fas ligand include: A) Cells of the blood-testes barrier. B) Cells lining the placental capillaries. C) CD4+ T cells. D) Cells of a metastatic carcinoma.

C - The question stem tells us that fas ligand serves a cytotoxic role, in other words, it induces the death of cells that it contacts and binds. Unlike CD8+ cells, CD4+ T cells do not have substantial cytotoxic activity. Although they do express fas, they have no reason to express Fas, they have no reason to express Fas ligand. A: Cells of the blood testes barrier express fas ligand in order to promote apoptosis and lymphocytes that threaten to infiltrate the immunologically privileged testes. Exposure of the testes to immune cells result in significant inflammation upon recognition of sperm, which are immunologically foreign. B: The placenta separates the maternal from the fetal circulation. It would make perfect sense for cells lining this barrier to express Fas ligand to prevent maternal immune cells from reaching the fetus, which is immunologically foreign. D: Metastatic cancer cells frequently express fas ligand. They use it to induce apoptosis in immune cells that would otherwise clear them from the body and prevent further cancerous growth.

According to the passage, the cagA gene (which leads to inflammation) product will cause: A) The disruption of host cell enzymatic activity. B) The disruption of host cell protein synthesis. C) The movement of leukocytes into mucosal tissue. D) The vasoconstriction of arterioles in the mucosal layer.

C - Thus, the cagA gene product triggers the movement of leukocytes into the mucosal tissue—because leucocytes gravitate toward an inflammation.

Which of the following factors would be most likely to cause acetic acid to completely dissociate in aqueous solution? A) Higher temperatures, which increase the pKa of the acid B) Enzymes that catalyze the forward reaction C) Continuous addition of acetic acid to the solution D) Continuous removal of protons from the solution

D - In acid dissociation reactions, aqueous H+ is a product. Le Châtelier's principle states that removing one or more products will cause equilibrium to shift toward the product side of the reaction. In other words, if protons are continually removed from the solution, acetic acid will continue to dissociate. A: Increasing the pKa of a species implies that it becomes a weaker acid. Weak acids do not completely dissociate in solution. B: Enzymes allow a reaction to proceed faster, but they do not affect equilibrium concentrations. Also, enzymes by definition catalyze both the forward and reverse reaction of a particular process. C: This would force the equilibrium toward the product side of the reaction, causing more H+ to be present in the solution. However, it would not cause the acetic acid to completely dissociate.

Of the following tissues, which is NOT derived from embryonic mesoderm? A) Circulatory B) Bone C) Dermal D) Nerve

D - Nervous tissue arises developmentally from ectoderm, not mesoderm. Thus, D is the best answer.

If the DNA of a representative species from each of the major kingdoms was examined, the sequences coding for which of following would be expected to be most similar? A. Photosynthesis B. Cholesterol synthesis C.Protein modification D. DNA synthesis

D - Not all kingdoms utilize photosynthesis or cholesterol synthesis. Protein modification is something that is quite unique to each phylum or even species, since it is one of the major ways in which species tailor protein function to their needs and create additional protein diversity from the limited set of genes they contain. DNA sequences that are common among different species, phyla, or even kingdoms are called conserved sequences. Conserved sequences tend to remain that way due to the fact that they code for a vital function that is common among disparate species.

Which of the following processes is involved in the motor-evoked potential elicited by transcranial magnetic stimulation? I. Na+ influx II. Ca2+ release III. Cl- efflux A) I only B) II only C) II and III only D) I and II only

D - Paragraph 1 states that the TMS pulse causes depolarization of the motor cortex and an electrical and motor response in the muscle tissue. This indicates that the action potential travels from the brain to the muscle, where it causes a muscle response (e.g., contraction). With this information in mind, let's begin this question by examining RN II. Muscle activation does require release of calcium from the sarcoplasmic reticulum to cause a mechanical response in the muscle, so RN II is correct (II). Since II is true, we can eliminate choice A. Move onto RN I, which is also true; depolarization (shown below in the context of the entire action potential) is characterized by a rapid influx of Na+ into the neuron (I).

To confirm that a protein has a silent mutation, you use: A) One dimensional gel electrophoresis B) Two dimensional SDS PAGE C) Western blot D) Sanger sequence

D- The only DNA analysis technique that shows the sequence of the associated gene

Cysteine versus methionine

Disulfide bonds are formed between the terminal tile groups of 16 residues. These bonds are important in the maintenance of the proteins tertiary structure, and then keratin, the larger the number of disulfide bond, the curlier the hair. Reducing these bonds breaks them into freestyle groups and relaxes the hair. Methionine does contain sulfur, I cannot form disulfide bond with either cysteine or other methionine residues

An individual is experiencing symptoms related to the lack of a specific protein. However, genetic analysis using a cDNA library finds no mutation in the gene encoding this protein. If a mutation does indeed exist, it would most likely be found: A) In the promoter region of the gene of interest B) In an intron within the gene of interest C) Downstream from the gene of interest D) In a gene coding for RNA polymerase

A - A mutation in the promoter sequence would prevent initiation of transcription entirely, resulting in a total lack of the relevant protein b. since intron sequences are spliced out during post-transcriptional modification, a mutation there would not affect the protein c. a mutation downstream a gene would likely have no effect; promoters and binding sites are all upstream of the gene of interest d. an error in RNA polymerase would affect transcription of every gene would be lethal to the patient

An asthmatic reacting to cat dander presents at the ER and the doctor orders an inspiratory chest X-ray. Which of the following is the X-ray most likely to look like during this asthma attack? A) Hyperexpanded lung volumes B) Normal lung volumes C) Areas of pus, derived from dead neutrophils and bacteria D) Diminished lung volumes

A - According to the passage, a major problem asthmatics have during an attack is air trapping. The smooth muscles of their bronchioles are hyperresponsive to triggers (like cat dander) and clamp down. Along with bronchiole constriction, inflammatory cells clog the airway with mucus. Thus, when asthmatics try to exhale, their airways have a smaller diameter, and they are forced to push harder with the diaphragm and accessory muscles to get the air out. The fast rush of air through the constricted bronchioles is responsible for the wheezing sound heard during an asthma attack. Therefore, the lungs of an asthmatic are hyperexpanded (choice A is correct and choices B and D are wrong). Choice C would be a good answer if the cause of the asthma attack was an infection. However, the question stem states that the cause was an allergen (cat dander), so no infection (and therefore no pus) should be seen.

Lymph is driven through the lymphatic vessels with the help of: A) One way valves that prevent fluid backflow. B) The motion of beating cilia that push the fluid along. C) High systolic pressure in the parallel arterial circuit. D) The collective motion of highly motile leukocytes in the fluid.

A - the lymphatic system exists under very low pressure, similar to the Venus system. One way valves are present to prevent backflow. C- The arterial circuit does not run parallel to the lymphatic vessels.

Which of the following enzymes is not found in the mitochondria? A) Hexokinase. B) Pyruvate dehydrogenase. C) Citrate synthase. D) Succinate dehydrogenase.

A -Hexokinase is the first enzyme to catalyze a step of glycolysis, which occurs in the cytoplasm. B: Pyruvate dehydrogenase converts pyruvate into a acetyl coA. This process occurs in the mitochondria. C,D: Both of these enzymes are found in the citric acid cycle which takes place in the mitochondria.

Membrane Bound Organelles

Arf1 is involved in the transport of proteins that need to be modified. The usual process to produce these proteins follows these steps: cytoplasm->endoplasmic reticulum->Golgi apparatus->cell membrane. As Arf1 is the protein that regulates retrograde movement from the Golgi, the vesicle will move back from the Golgi apparatus to the endoplasmic reticulum.

The single diploid cell formed when an ovum is fertilized by a spermatozoon is known as a(n): A) Embryo B) Zygote C) Morula D) Gastrula

B - A zygote is formed by the fusion of gametes (one sperm and one ovum) as both gametes are haploid, fusion results in a single diploid cell a- an embryo is a multicellular organism with developing organ systems. c - four cell divisions occur with no significant growth, with the resulting ball of cells is the morula d- a gastrula is a collection of cells containing three germ layers; the question is asking about an earlier stage of development

Pyruvate kinase, which converts PEP to pyruvate is regulated by: I. ATP. II. Acetyl Co-A. III. Oxaloacetate. IV. Fructose 1-6 biphosphate. A) I only B) I and II C) I, II, and IV D) I, II, III, and IV

B - ATP and acetyl-coAare regulators of this final step of glycolysis.

Introduction of which amino acid substitution would result in the largest decrease in the entropic penalty associated with a protein folding into its native conformation? A) Ile to Asp substitution at a buried site B) Leu to Thr substitution at a surface-exposed site C) Gly to Pro substitution in a flexible loop D) Arg to Tyr substitution at a surface-exposed site

B - When Leu, a hydrophobic amino acid, is located at a surface-exposed site of proteins, it is in contact with water molecules and, therefore, water molecules must organize themselves into a specific arrangement (entropic penalty) to live with this fact. A substitution of Leu to Thr, which is a polar and hydrophilic amino acid, reduces drastically this entropic penalty because water molecules are now in contact with a hydrophic amino acid. C: Proline has a fused ring side chain, making the amino acid sequence in its vicinity very rigid. In contrast glycine has a small side chain and would be very flexible in a flexible loop. Therefore, this substitution will increase, not decrease, the entropic penalty associated with a protein folding.

The correct sequence of DNA packaging in chromosomes, from the smallest component to the most complete form, is: A) double stranded DNA -> chromatin -> histones -> nucleosomes B) double stranded DNA -> histones -> nucleosomes -> chromatin C) chromatin -> double stranded DNA -> histones -> nucleosomes D) chromatin -> double stranded DNA -> nucleosomes -> histones

B - chromatin is fully packaged DNA

Which of the following characteristics clearly marks fungi as eukaryotes? A) They have cell walls. B) They contain ribosomes. C) They contain mitochondria. D) They exhibit sexual reproduction.

C

What aspects separate single-crossover events from double-crossover events? A) Single-crossover events result in one-way displacement of chromosomal content from one chromosome to another, while double-crossover events always reverse this one-way displacement, resulting in chromosomes identical to the pre-crossover chromosomes. B) Single-crossover events occur during mitosis when a cell splits into two cells, while double-crossover events can only occur during meiosis when a cell splits into four cells. C) Single-crossover events affect only the ends of chromosome arms, while double-crossover events can affect segments in the middle of chromosome arms. D) Single-crossover events only affect one arm of each chromosome, while double-crossover events affect two arms of each chromosome.

C - A double-crossover event is one in which chromosomal arms of homologous chromosomes cross over in two different places along the arm. This results in a section in the middle of each chromosome being exchanged.

If a filamentous body were isolated from an infected individual and injected into an uninfected healthy individual what antibodies would be formed? A) Anti-core IgM B) Anti-eAg IgG C) Anti-sAg IgM D) Anti-sAg IgG

C - A filamentous body is described in the passage as lacking core and genetic elements, thus anti-core antibodies would not be formed (choice A is wrong). The passage further states that IgM is the first line antibody and is replaced by IgG for long term protection. Since the filamentous bodies are being injected into an uninfected individual, the first antibody to be made will be IgM (choices B and D are wrong and choice C is correct).

In the macronucleus, the genes for rRNA are located extrachromosomally. This suggests that the rRNA genes are: A) Nonlinear. B) Nonfunctional. C) Self-replicating. D) Rearranged.

C - Genes are strands of DNA, so this must mean that they are strands of DNA that are not part of the cell's chromosomes. These strands still must self-replicate just as the chromosomes do if they are to be passed from one generation to the next, so choice C is the correct answer. All nucleic acids are a linear arrangement of the component nucleotides, so choice A is incorrect. The rRNA genes must be functional or the organism could not produce protein, so choice B is incorrect. Finally, the fact that they are extrachromosomal gives no clue to their arrangement, so choice D can also be rejected.

Patients with excess fat are more likely to require larger therapeutic doses of which vitamin? A) Vitamin B1 B) Vitamin C C) Vitamin D D) Vitamin B3

C - The lipid-soluble vitamins are A, D, E, and K. The water-soluble vitamins are B and C. Greater amounts of subcutaneous fat sequester more of the lipid-soluble vitamins and lower their release into the circulation. Thus, excess fat increases the initial dose of vitamin required to achieve a particular effect.

Troponin isoenzymes are used as an alternative biomarker in the diagnosis of heart attacks. In which of the following muscle types does the troponin complex function in contraction? I. Skeletal muscle II. Smooth muscle III. Cardiac muscle A) I only B) I and II only C) I and III only D) I, II, and III

C - Troponin is a complex of three proteins (troponin I, troponin C, and troponin T) required for muscle contraction in skeletal muscle and cardiac muscle, but not smooth muscle (I and III).

Uracil is usually found in: I. tRNA. II. ribosomes. III. single-stranded DNA. A. I only B. II only C. I and II only D. I, II, and III

C - Uracil is found in any structure made of RNA. Both tRNA and ribosomes are made of RNA (I and II). Single-stranded DNA has thymine rather than uracil (III).

Which of the following are true statements regarding genomic and cDNA library is? I. A genomics library contains at least one copy of all of the sequences in an organisms genome. II. CDNA library contains only expressed sequences of a genome. III. A cDNA library contains copies of DNA that was transcriptionally active in the cell at the time the library was made. A) I only B) II only C) I and II only D) I, II, and III

D

Which of the following are true of eukaryotic mRNA before it leaves the nucleus? I. A 3' cap is added II. A 5' cap is added III. Exons are spliced out of the sequence IV. Introns are spliced out of the sequence A) I and III B) I and IV C) II and III D) II and IV

D - A 7 methylguanylate cap is always added to the g' end of pre mRNA to protect it from degradation; introns are excised through the use of the spliceosome I. Its a poly a TAIL not cap III. Exons consist of DNA that is able to be expressed, so they remain

To perform a successful PCR reaction, all of the following reactants must be present EXCEPT: A) Forward and reverse primers B) Taq polymerase C) A magnesium containing buffer D) Dideoxynucleoside triphosphate (ddNTPs)

D - A ddNTPs lacks an OH group at both its 2' and 3' positions. Since DNA polymerase requires a free 3' OH to continue replication, ddNTPs serve as inhibitors of the chain elongation, terminating replication. Primers bind to the gene of interest and signal where DNA polymerase should attached, required for both strands of the dsDNA molecule; Heat resistant tag is used in PCR; Magnesium serves as a cofactor for Taq and stabilizes the negative charges on the two strands, which would otherwise repel each other

The peptide bond that forms the backbone of proteins is especially stable because it: A) Consists of a triple bond, which is significantly stronger and more stable. B) Is a carboxylic acid derivative. C) Would result in proteins that denatured easily if it were unstable. D) Exhibits resonance stabilization.

D - A peptide is an amide linkage between the amine group of one amino acid and the carboxylic acid group of another. Amides are stable because they exhibit resonance stabilization between the lone pair on the nitrogen and the double-bond in the carbonyl group

How did the positive and negative ends of actin different? A) The monomers on the positive end are bound to ATP, while those on the negative and are bound to ADP. B) The positive and actin monomers have a greater binding affinity for each other than do those on the negative end. C) The positive and is always closer to the outside of the cell. D) Both A and B are correct.

D - Actin strands polymerize with the positive end and depolymerize of the negative end. In polymeric form, ATP bound action (found at the positive end) has a much higher affinity than the ADP bound form present at the negative end. Therefore, both A and B are true.

When a striated muscle cell metabolizes glucose in the complete absence of O2, which of the following substances is NOT produced in a significant amount? A) Pyruvic acid B) Glucose-6-phosphate C) Lactic acid D) Acetyl-CoA

D - If oxygen is scarce or absent, the product of glycolysis will be transformed into lactic acid instead of acetyl-CoA. Thus, the cell will produce less acetyl-CoA. A: Pyruvic acid is one product of glycolysis. B:Glucose-6-phosphate is a product of glycolysis. C: Lactic acid is a product of pyruvic acid fermentation, which is a process that takes place when the levels of oxygen are low.

Phenylketonuria is a genetic disorder caused by a mutation in the gene for the enzyme phenylalanine hydroxylase, which eliminates its enzymatic activity. Could an antisense drug help individuals with this disorder? A) Yes, if it binds to the mRNA of the phenylalanine hydroxylase gene and prevents its translation B) Yes, if it is incorporated into the chromosome and prevents the expression of the phenylalanine hydroxylase gene C) No, because mRNA does not persist in the cytoplasm of the cell D) No, because blockage of phenylalanine hydroxylase gene expression will not remedy the original disorder

D - The question indicates that phenylketonuria (PKU) is a genetic disorder caused by a mutation in the gene for an enzyme, which results in the elimination of enzymatic activity. The question asks the examinee to consider whether an antisense drug could help individuals who have PKU. An antisense drug works to prevent the expression of undesirable genes but does nothing to remedy the problem of a gene that produces an ineffective product. The only way to cure PKU would be to add a gene or gene product that could lead to the production of effective enzymes to replace the ones that do not function correctly. D is the best answer because a blockage of gene expression would only prevent the production of defective enzymes but would not remedy the problem caused by the absence of effective enzymes. A and B are incorrect because preventing expression of an inactive protein is not a cure. C is incorrect because the stability of the mRNA is irrelevant to the situation.

All of the following chromatographic methods are used to purify substances except: A) Size exclusion chromatography. B) Affinity chromatography C) Ion exchange chromatography. D) Thin layer chromatography.

D - Thin layer chromatography is used for analysis not purification. Specifically, a characterize the samples by their RF value. It can also be used to identify compounds in a mixture and determine overall purity.

Tight junction between cells are not likely to be found in which of the following areas? A) The cerebral vascular epithelia B) The mucosa of the small intestine C) The glomerular capsules of the kidney D) The capillaries of the lung

D - Tight junctions are necessary in all epithelial cell layers that have functions that depend on preventing free diffusion between the sides of the layer. such cells use transmembrane transport proteins to move molecules through the cell. in contrast, the free exchange of gas molecules is the entire purpose of lung tissue, lung capillaries are therefore extremely leaky and contain no tight junctions A,B,C,: the blood brain barrier, the selective transport of nutrients, and the exclusion of filtrate from the kidney ECM all rely on the presence of tight junctions to form a seal against diffusion

The active site of an enzyme binds positively charged molecules, but a researcher discovers that the enzyme is inhibited when it interacts with a certain large anion. This molecule likely: A) Dimerizes the enzyme. B) Competitively bind the active site. C) Allosterically by the active site. D) Allosterically by the regulatory site.

D - from the question stem, it is known that the negatively charged molecule cannot bind at the active site. Description of this interaction is characteristic of allosteric regulation.

The main role of serum albumin, a solute 65 kDa, protein is to: A) Bind to and carry oxygen via its many heme groups B) Bind to and carry carbon dioxide via its many heme groups C) Maintain plasma osmolarity, causing fluid to diffuse from the tissues into capillary beds D) Maintain plasma osmolarity, causing fluid to diffuse from the tissue into all blood vessels

D- All blood vessels - arteries, arterioles, veins, venues, and capillaries - contain a thin inner lining composed of endothelial cells. This lining forms the interface, or immediate barrier, between blood and the remainder of the vessel, except in the capillaries which are made of endothelium alone. Lympathic vessels also have this

Which of these molecules is produced by the Dicer enzyme and involved in RNA silencing? A) tRNA B) rRNA C) miRNA D) siRNA

D- si RNA is a cals of dsRNA that is crucial in RNA interference; siRNA molecules are produced from small hairpin RNAs and dsRNAs in a process catalyzed by the Dicer enzyme tRNA - translation/amino acid recognition rRNA - component of ribosomes/protein synthesis miRNA- involved in RNA silenced, they are produced by RNA polymerase II

HPLC (high performance liquid chromatography)

HPLC is a powerful form of liquid chromatography. Unlike other forms of liquid chromatography, which rely on gravity, HPLC utilizes high pressure to force a sale but there was smaller, more tightly packed stationary phase. This process is a more effective purification and analytical method than many traditional forms of chromatography. The mobile phase and HPLC is no different from the mobile phase is used in other forms of chromatography. Hexane toluene, and water, are common solvents.

Erythrocytes

Red blood cells do not contain a nucleus or mitochondria and therefore cannot use the citric acid cycle or electron transport chain to create ATP. Neurons, epithelial tissues, and T cells do you have mitochondria and can therefore utilize aerobic respiration.

The RNA induced silencing complex is a ribonucleoprotein that uses dsRNA fragments to target mRNA transcripts. This process is most accurately characterized as: A) Complementary basepairing B) RNA interference c) mRNA targeting D) Intragenic RNA silencing

b

Would an increase in the level of plasma aldosterone be expected to follow ingestion of excessive quantities of NaCl? A) No; aldosterone causes Na+ reabsorption by kidney tubules. B) No; aldosterone causes Na+ secretion by kidney tubules. C) Yes; aldosterone causes Na+ reabsorption by kidney tubules. D) Yes; aldosterone causes Na+ secretion by kidney tubules.

A - Aldosterone, which is produced by the adrenal cortex, causes Na+ reabsorption by kidney tubules. Such a mechanism decreases Na+levels in the urine. The steroid aldosterone does not cause Na+ secretion into the urine. Because ingestion of excessive NaCl would trigger Na+secretion into the urine, plasma-aldosterone levels would not increase. Rather, the body would rely on those homeostatic mechanisms that excreted the excess Na+.

How does a codon recognize the correct anticodon? A) The anticodon binds to the codon via corresponding RNA sequences B) The anticodon binds to the codon via a lock and key mechanism C) The anticodon binds to the codon via corresponding DNA sequences D) The anticodon binds to the codon via accessory proteins attached to the RNA

A - An anticodon is part of a tRNA molecule; it is able to bind to a corresponding codon because it possesses a complementary RNA sequence

Starting from its location of synthesis, list the structures contacted by bile as it travels. A) Liver, gallbladder, cystic duct, common bile duct, duodenum. B) Liver, common bile duct, gallbladder, cystic duct, duodenum. C) Common bile duct, liver, cystic duct, gallbladder, stomach. D) Gallbladder, common bile duct, liver, cystic duct, stomach.

A - Bile is produced by the liver, then travels to the gallbladder, where it is stored until needed. Once the gallbladder receives a signal to release bile, the substance travels down the cystic duct and the common bile duct which is shared by the liver, then into the duodenum.

Cell differentiation is mediated primarily by: A. Gene expression levels. B. Differing genetic material. C. Location within an organism. D. DNA ethylation.

A - Cell differentiation occurs primarily through different gene expression levels. B: Nearly all cells share the same genome, so differences in the genetic material isn't a realistic explanation. C: Location within the body is not responsible for cell differentiation. A skin cell transplanted to the liver, for example, will not become a liver cell. D: It is DNA methylation, not ethylation, which plays a major role in gene expression and, consequently, cell differentiation.

In oxidative phosphorylation, cytochrome c acts as: A) 1-electron carrier. B) 2-electron carrier. C) 3-electron carrier. D) 4-electron carrier.

A - Cytochrome c is a heme protein that cycles between a ferrous (Fe+2) and ferric (Fe+3) state during oxidative phosphorylation. Therefore, only a single electron is transferred in the process.

The passage states that HDAC1 catalyzes the production of TGFβ and acts to remove acetyl groups from lysine residues of histones. The catalyst for TGFβ production most likely plays a direct role in: A) Gene silencing. B) Ribosomal inactivation. C) Transfer of an acetyl group from acetyl-CoA. D) Gene activation.

A - DNA is wrapped around histone proteins, and DNA expression is regulated by the addition and removal of acetyl groups. Deacetylation attracts DNA to histones more tightly, inhibiting transcription. In summary, histone acetylation generally serves to increase gene expression, while deacetylation, along with DNA methylation, acts to silence gene expression.

Molecular biologist runs an SDS-Page on protein A, using mercaptoethanol in the preparation of the protein sample. Protein A is known to possess quaternary structure. What will be the likely result after staining with Coomassie blue? A) Multiple bands that are further along the gel relative to protein A's found in a native electrophoresis. B) Multiple bands that are closer to the start of the gel relative to protein A's band and a native electrophoresis. C) A single band towards the start of the gel. D) A smear traversing the length of the gel.

A - Denaturation via SDS and mercaptoethanol effectively removes all forces that hold together secondary, tertiary, and quaternary structure. Therefore, the various subunits of protein A with separate individually as they migrate along the gel. In a native gel, the protein is not teenager, and protein it would migrate as a single, heavier molecule. This would result in slower travel.

Stimulation of the iris dilator muscle is a result of activation of: A) Sympathetic motor neurons. B) Parasympathetic motor neurons. C) The fifth cranial nerve. D) Sympathetic sensory neurons.

A - Dilation of the pupils is a classic fight-or-flight response. The fight-or-flight response is part of the sympathetic nervous system.

How does secretion differ from filtration? A) Secretion refers to the transport of specific toxins and protons into the lumen of the nephron, while filtration relates to the original movement of fluid and solute into the Bowmans capsule. B) Secretion refers the transport of specific toxins and protons into the lumen of the nephron, while filtration relates to the original movement of fluid and solute into the glomerulus. C) Secretion permits a passage of all molecules except protein, while filtration is a specific process it only allow certain molecules to enter the nephron. D) Secretion takes advantage of passive diffusion, while filtration is accomplished mainly via secondary active transport.

A - Filtration, secretion, and reabsorption are extremely easy terms to confuse. Remember, filtration refers to the general movement of plasma from the glomerulus, a tight network of capillaries, and through the entrance of the nephron at the Bowmans capsule. This process is extremely general, with virtually any molecule smaller than protein allowed passage into the filtrate. B: Filtration involves movement from the glomerulus into Bowmans capsule, not the other way around. C: This is the opposite of the correct answer. D: This choice, too, is the virtual opposite of the correct answer. Filtration is accomplished mainly by physically forcing fluid to enter the nephron. As such, it is a passive process that relies on diffusion.

A follow-up experiment assaying for cell cycle arrest with radiolabeled [3H]-thymine indicated that CRC157 cells transfected with pC27-53 did not incorporate [3H]-thymine during DNA synthesis. Based on this information, at what regulation point in the cell cycle does WT p53 arrest cell growth? A) G1/S B) S/G2 C) G2/M D) M/G1

A - Given that uracil, not thymine, is incorporated during transcription, the radiolabeled [3H]-thymine would only be incorporated during DNA (and not RNA) synthesis. DNA synthesis only occurs during the S phase of the cell cycle. Therefore, in order for p53 to prevent the uptake of [3H]-thymine, the protein must arrest the cell cycle prior to the S phase at the G1/S regulation checkpoint.

The symptoms of botulinum took 12-18 hours to be observed in the patient. How is this best explained? A) Growing bacteria produce the toxin. B) Botulinum toxin is immediately filtered by the kidneys and excreted. C) Toxins can be absorbed through mucous membranes or respiratory tract. D) Most of the toxins are polar and they take time to cross the nuclear membrane.

A - If growing bacteria produce the toxin, then it will take time for the bacteria to generate sufficient quantities of the toxin to cause illness. The toxin has its effect on the neuromuscular junction and does not cross the nuclear membrane.

Which of these procedural measures would contribute most to optimal PCR function in a high-yield of the desired product.? A) Selection of a primer sequence that does not complement itself. B) Addition of metal chelating agents to the reaction solution. C) Use of a DNTP solution with very high pyrimidine concentrations and relatively low concentrations. D) Use of a DNA sample with an extremely high molecular weight.

A - If primers are complementary to each other primer nucleotides manual with one another to form a small useless timer. Selection of primer sequences that do not exhibit this characteristic will decrease the chances of promoting the side reaction, ultimately optimizing the desired PCR. B: Since magnesium is an essential component of a PCR solution, a metal chelating agent would likely reduce reaction rate. C: Although hydrogen bonding favors the addition of complementary nucleotides, errors may occasionally occur. Relatively high pyrimidine concentration and solution increases the probability that a pyrimidine will be wrongly incorporated into the growing strand, simply due to the law of mass action. Since tack polymerase has limited proofreading ability, this error is unlikely to be resolved. Optimally, PCR involves the use of equal concentrations of all four nucleotides. D: PCR is error prone and works best when used to amplify DNA strands that are not particularly large.

Which of the following statements is/are true with regard to the mechanics of skeletal muscle contraction? I. Reduced ATP levels could result in high intracellular calcium and persistent contraction II. Calcium binds to calmodulin and allows actin and myosin to interact III. The shorter the sarcomere length, the stronger the contraction A) I only B) I and II only C) II and III only D) I, II, and III

A - Item I is true: in the absence of ATP, not only will the myosin and actin filaments be unable to disconnect, Ca2+ active transporters will not be able to return Ca2+ to the sarcoplasmic reticulum (choice C can be eliminated). Item II is false: calcium binds to troponin (not calmodulin) in skeletal muscle (choices B and D can be eliminated and choice A is correct). Note that Item III is false: the length-tension relationship states that there is an optimum sarcomere length to ensure the most forceful contraction. While it is true that shorter sarcomere length translates to a stronger contraction up to a point, too much overlap causes the actin and myosin filaments to bump together and reduces the strength of contraction.

How does kinesis travel across a microtubule? A) The hydrolysis of ATP triggers a conformational change in the protein, after which the replacement of ADP with ATP triggers another structural change and completes the cycle. B) The kinesin binds the microtubule and travel along with it as a microtubule grows and shrinks. C) The kinesins only have affinity for Alpha tubulin and move down to microtubular as they switch between alpha tubulin monomers. D) The hydrolysis of water triggers a confirmational change in the protein that allows it to travel down to my car to go.

A - Kinesins have two conformations, one when bound to ATP and a second when bound to ADP. By hydrolyzing ATP into ADP, the Chinese and can move down to microtubule. Then, replacing ADP with ATP causes the Chinese and to reach out further out on the microtubules and the cycle continues

When selecting a buffer for PCR, it is most important to ensure the solution contains: A) Magnesium cations. B) Free ribonucleotide triphosphates. C) Hydroxide anions. D) Pyrophosphate anions.

A - Magnesium is a cofactor for taq DNA polymerase. This ion aids in the coordination of the pyrophosphate leaving group, facilitating phosphodiester hydrolysis of the subsequent addition of incoming nucleotide triphosphates. B: PCR requires free deoxy nucleotide triphosphates DNTPS. Free dNTPS are a necessary in such a procedure. C: Hydroxide ions would serve no purpose during PCR. They would probably hamper proteins function and caused DNA denaturation due to the resulting basicity of the solution. D: Pyrophosphate as a side product of DNA polymerization. Not only is this a necessary for a PCR procedure, it's presence in solution may actually reduce the rate of the forward reaction..

Unlike the cells from which human organs are composed, the cell of a unicellular organism such as algae: A) Has a genome where nearly all material codes for protein. B) Typically utilizes mitosis for cellular division. C) Can perform catabolic reactions to gain energy from macromolecules. D) Contains membrane-bound organelles to execute cell functions.

A - Nearly 95% of the human genome does not code for proteins or RNA. In contrast, the genomes of both prokaryotes and unicellular eukaryotes largely lack introns. In these organisms, most genetic material does code for protein products. B, C, D: This is a characteristic of human cells. The question asked for a trait that is unique to unicellular organisms.

Which of the following techniques would be LEAST likely to detect a Robertsonian translocation? A) Southern blot B) Routine karyotype C) High-resolution karyotype D) Fluorescent in situ hybridization of chromosomes

A - Southern blots are used to analyze extracted DNA for a potential DNA segment and would not provide detailed information with regard to chromosome structure. In other words, you would know if a gene was present, but would not know if it was on the correct chromosome or not (choice A would be the least likely to detect a translocation and is the correct answer choice). Karyotyping (both routine and high-resolution) is capable of detecting a missing chromosome (choices B and C can be eliminated). Fluorescent in situ hybridization uses fluorescent DNA probes to bind to chromosomes and could detect a translocation by detecting a gene on an unexpected chromosome (choice D can be eliminated).

The cutting of introns and the splicing associated with the expression of the Factor VIII gene occur during which of the following steps in the protein-synthesis process? A) DNA → mRNA B) DNA → tRNA C) mRNA → tRNA D) tRNA → protein

A - Splicing is part of the post-transcriptional process: DNA is first transcribed in primary transcript and then modified to become mRNA. B:DNA coding for Factor VIII is not transcribed in tRNA. C: The mRNA coding for Factor VIII is not modified into tRNA. D: tRNA is not translated into protein.

In which organelle of a eukaryotic cell is the pyrimidine uracil, as part of uridine triphosphate (UTP), incorporated into nucleic acid? A) The nucleus B) The Golgi bodies C) The ribosomes D) The endoplasmic reti

A - The nitrogenous base, uracil, combined with the sugar ribose and phosphate makes up the nucleotide uridine. It is found in RNA, but not in DNA. The corresponding DNA nucleotide is thymine. Uridine is incorporated into RNA in the nucleus where transcription of DNA into RNA takes place. RNA is manufactured in the nucleus from a DNA template. Therefore, the correct answer is answer choice A. RNA is necessary for protein synthesis by free ribosomes and those attached to endoplasmic reticulum, but it is not synthesized there, so answer choices C and D are incorrect. Proteins are packaged for export in the Golgi apparatus, but it too takes no part in RNA synthesis, so answer choice B is incorrect.

Which of the following is likely true of the pentose phosphate pathway? A) It is more active in adipose tissue than in muscle tissue. B) NADPH is produced and can be used in the electron transport chain and oxidative phosphorylation to generate ATP. C) It is a series of isomerizations of six-carbon sugars. D) The inhibition of carbamoyl phosphate synthase by purine nucleotides helps to regulate the production of UTP and CTP.

A - The pentose phosphate pathway replenishes NADPH, which the passage states is used by the cell in biosynthetic pathways such as fatty acid biosynthesis. This is more likely to happen in adipose (fat) tissue than in muscle tissue (choice A is correct). NADPH cannot, however, be used to generate ATP via the electron transport chain and oxidative phosphorylation; NADH is needed for that pathway (choice B is wrong). The pathway produces ribose, a 5-carbon sugar from glucose, a 6-carbon sugar; this involves a decarboxylation, not just an isomerization (atomic rearrangement, choice C is wrong). Lastly, the passage states the carbamoyl phosphate is inhibited by UTP, which is a pyrimidine, not a purine (choice D is wrong).

Which of the following statements is true regarding peptide bonds found in the CK subunits analyzed? A) They possess partial double bond character. B) They are ionized at physiological pH. C) They occur most commonly in the cis configuration. D) They are cleaved by high urea concentrations.

A - The peptide bond displays partial double bond character because of the delocalization of electron density from the peptide (amide) nitrogen through the peptide carbonyl carbon and onto the peptide carbonyl oxygen. As such, the peptide bond exhibits resonance stabilization, as shown below. The carbonyl oxygen and amide nitrogen of the peptide linkage are stable and not ionizable at physiological pH. For steric reasons, the peptide bond is normally found in the trans, not the cis, configuration. While urea affects higher levels of protein structure, it does not disrupt peptide bonds, which are generally liable only to strongly acidic conditions.

In eukaryotic cells, the process of incorporating uridine nucleotides into nucleic acid polymers occurs in which of the following structures of the cell? A) Nucleus B) Lysosome C) Ribosome D) Golgi body

A - The question asks the examinee to identify the cellular structure in eukaryotic cells in which the process of incorporating uridine nucleotides into nucleic acid polymers occurs. Uridine nucleotides are incorporated into RNA, and the question is therefore asking where transcription occurs. In a eukaryotic cell, transcription occurs in the nucleus (A). Thus, A is the best answer.

The macronuclei of the asexual progeny in Tetrahymena and the cytoplasm of the ova-producing cells of female vertebrates share a common feature in that both: A) Undergo uneven division. B) Contain uneven amounts of nuclear material. C) Regulate their contents by adding or skipping an S phase. D) Are apportioned at mitosis.

A - When a vertebrate oocyte divides in meiosis the cytoplasm is distributed to the two daughter cells. Most of it goes to the daughter cell destined to be the ovum. The other daughter cells, the polar bodies, are cast off with little cytoplasm. The amount of nuclear material in each, however, is the same, so choice B is incorrect. The distribution of cytoplasm is uneven, as is the distribution of macronuclear DNA during the amitotic cell division in Tetrahymena.

Dendritic cells are often credited as being the link between the innate and adaptive immune systems. The cells are capable of sampling the membranes of somatic cells using a process called nibbling. The denture excel then carries a cellular sample to the nearest lymph node and present the antigen to another immune cell, in a process that involves direct membrane contact and utilize MHC. Assuming the sample sale was subject to a novel intracellular infection, the dendritic cell most likely interacts with: A) A helper CD4+ T cell B) A macrophage. C) A memory B cell. D) A cytotoxic CD8+ T cell.

A - dendritic cells present antigens, in the context of MHC, to immature CD4 T cells. The T cell will become activated if it has affinity for the antigen/MHC presented by the dendritic cell. And activated CD4 T cells will produce a unique cytokine profile to regulate other immune cells and facilitate an effective response to the pathogen. B- Under some circumstances, macro phages can present antigens in the context of MHC. However, only T cells display the receptor capable of recognizing MHC. Macro phages are unable to interact with dendritic cells. C: B cells recognize intact anti-genic domains. They do not recognize antigen fragments in the context of MHC. Furthermore, a memory visa would be unlikely to recognize an antigen derived from an unfamiliar pathogen. D: although cytotoxic T cells are capable of recognizing MHC loaded with antigen, they respond by perforating the presenting cell. While this is an effective way to kill an infected somatic cell, it would be counterproductive to the object of the dendritic cell, which is to inform the immune system of a foreign invader.

Of the following, which accurately describes a difference between glycolysis and gluconeogenesis? A) Glycolysis can occur in all cells while gluconeogenesis cannot. B) Glycolysis and gluconeogenesis occur in different parts of the cell. C) Only gluconeogenesis requires ATP to function. D) Glycolysis can occur in the absence of oxygen, while gluconeogenesis cannot.

A - glycolysis occurs in all cells well gluconeogenesis only occurs in the liver and cortex of the kidney.

In general, how does breathing function to maintain the balance of carbon dioxide and oxygen in the lungs and blood stream? A) breathing keeps the concentration of oxygen high in the alveoli to facilitate diffusion into the blood stream B) Breathing rate decreases with physical activity to provide oxygen to the muscles. C) Breathing removes carbon dioxide from our system, that's reducing the pH of our blood. D) The rate of breathing is regulated depending on the body's demand for oxygen or carbon monoxide.

A - simple diffusion is the mechanism by which both oxygen and carbon dioxide move from the bloodstream into the alveoli and back. The movement of these gases is based on their concentrations in one location relative to the other.

Increasing plasma concentration of aldosterone is most likely to be followed by which of the following? A) Increased water reabsorption through increased aquaporin channels in the collecting duct B) Increased sodium reabsorption in the distal tubule C) Decreased water reabsorption in the collecting duct D) Decreased plasma calcium concentration

B - Aldosterone is released from the adrenal cortex in response to low blood pressure. Its primary function is to increase sodium reabsorption in the distal tubule and collecting duct. Aldosterone upregulates the sodium-potassium pumps along the lining of the nephron, pumping three sodium ions OUT of the nephron lining (and toward the blood) for every two potassium ions it pumps IN (toward the nephron and away from the blood). Since we have a net solute movement out of the nephron, aldosterone also increases the gradient that favors water reabsorption.

The scientist wanted to use antibody B clinically (to treat humans), but this proposal was rejected. Which of the following is the most logical reason for the rejection? A) Because the antibody was generated in the mouse, it can never be used in humans. B) Because the antibody was generated in the mouse, repeated usage in the same patient would elicit the production of human anti-mouse antibodies. C) Because the antibody was generated in the mouse, it will not recognize human antigens. D) Because the antibody was generated in the mouse, it can only be used in vitro.

B - As the antibody was produced in another organism, several uses might trigger an immune response in the human individuals that would recognize the molecule as foreign. A: Antibodies generated in other organisms can be used in humans. For instance, this is how antibodies are generated against poisons and given to humans as a first response. C: Often antibodies generated by an organism cross react with other organisms. D: Antibodies generated by other organisms can be used in vitro as well as in vivo in humans if necessary and effective.

An operon is a functional group of genes that can be organized together. All of the following are regulatory sequences within the operon EXCEPT: A) The operator B) Protein coding genes C) The enhancer D) The promoter

B - Coding regions are the DNA segments that are directly transcribed into mRNA and then translated into protein products; a) during transcription, the transcription factors that regulate gene expression bind to the operator sequence c) the enhancer is the region where activators can bind to increase the rate of transcription d) the promoter is a regulatory sequence that initiates transcription

Some eukaryotic cells are covered with small ciliary projections used for absorption, while others contain larger flagella used for propulsion. These cellular structures are composed of: A) Microfilaments. B) Microtubules. C) Intermediate filaments. D) Myosin.

B - Eukaryotic cilia and flagella are composed of bundles of microtubules. Note that this differs from prokaryotic flagella, which are formed from the protein flagellin. A: Microfilaments are composed of actin and are found in the cytoplasm of eukaryotic cells, as well as in muscle. C: Intermediate filaments are less dynamic than actin or microtubular filaments. They are not involved in ciliary or flagellar structure. D: Myosin is present in muscle and aids in the process of contraction.

ACE inhibitors are a class of drugs frequently prescribed to treat hypertension. Captopril, a compound that is structurally similar to angiotensin I, was developed in 1975 as the first ACE inhibitor. When patients take Captopril, which of the following is true about the kinetics of their ACE? A) Vmax decreases, Km remains the same B) Vmax remains the same, Km increases C) Both Vmax and Km increase D) Both Vmax and Km remain the same

B - From the question text, it can be deduced that Captopril is a competitive inhibitor of ACE (since its structure is similar to that of the substrate, angiotensin I). Competitive inhibitors do not alter an enzyme's Vmax, since the active site can be saturated by increasing the substrate concentration (choices A and C can be eliminated). Competitive inhibitors do increase the apparent Km of the enzyme, because a higher substrate concentration is required to reach ½ Vmax (choice D can be eliminated).

Injection of insulin into the bloodstream is LEAST likely to result in which of the following? A) Increased glycogen synthesis B) Decreased lipid synthesis C) Increased esterification of fatty acids D) Decreased gluconeogenesis

B - Insulin is secreted in response to high blood sugar. If the body detects that there's plenty of blood sugar, then it would want to stop making more sugar, to store that sugar as glycogen, and to build up fatty acids into fats for storing up energy. The thing the body is LEAST likely to do is to stop storing up energy. In general, think of the function of insulin as causing the body to build up large molecules to store up energy (glycogen, lipids) and to stop the body from breaking down large molecules to provide energy.

Viral infection would lead to an increase in which of the following cellular pathways? I. Aerobic respiration II. Pentose phosphate pathway III. cAMP mediated protein kinase activation A) I only B) I and II only C) II and III only D) I, II, and III

B - Item I is true: viral infected cells are undergoing a fair amount of cellular activity, including DNA replication, RNA transcription, and protein synthesis, all of which contribute to an increased need for ATP, and thus oxidative respiration (choice C can be eliminated). Item II is true: the pentose phosphate pathway produces essential precursors for nucleotide synthesis, necessary for both DNA replication and RNA transcription. It's likely that the activity of this pathway would be increased during viral infection (choice A can be eliminated). Item III is false: cAMP mediated protein kinase activity is increased when G protein coupled receptors are bound and activated; there is no reason to assume this in viral infection (choice D can be eliminated and choice B is correct).

Which of the following proteins travel down microtubules towards the outside of the cell? A) Cadherins B) Kinesins C) Dyneins D) Actin

B - Microtubules originate from the MTOC, which is located close to the center of the cell. Because microtubules grow from the positive end, that end is located towards the periphery of the cell while the negative ends are found close to the center. Kinesins travel toward the positive end of microtubules, and therefore must carry cargo towards the outside of the cell. Dyneins carry cargo towards the center of the cell.

With which of the following processes is a double-stranded DNA break NOT typically associated? A) Recombination B) Mitosis C) Balanced Robertsonian translocation D) Unbalanced Robertsonian translocation

B - Mitosis does not typically involve double-stranded DNA breaks, although they can occur by chance (choice B is the correct answer). Recombination (crossing over) in meiosis begins with a double-stranded DNA break, followed by invasion of the opposite strand (choice A involves a double-stranded break and can be eliminated). Both balanced and unbalanced Robertsonian translocations must involve double-stranded DNA breaks in order for two chromosomes to exchange long arms (choices C and D can be eliminated). The only difference between balanced and unbalanced translocations is that in an unbalanced translocation some genetic information (DNA) is gained or lost.

The sequence of events in the human menstrual cycle involves close interaction among which organs? A) Hypothalamus-thyroid-ovary B) Hypothalamus-pituitary-ovary C) Pituitary-thyroid-ovary D) Pituitary-adrenal glands-ova

B - Neither the thyroid gland nor the adrenal glands are particularly involved in the menstrual cycle. The hypothalamus exerts control over the pituitary hormones involved in menstruation by secreting hormone-releasing factors into the pituitary portal circulation. The gonadotropic hormones FSH and LH produced by the pituitary and the ovarian hormones estrogen and progesterone all have a role in regulating the human menstrual cycle. Therefore, close interaction among the hypothalamus, the pituitary, and the ovary is necessary for the human menstrual cycle.

In mammals, which of the following events occurs during mitosis but does NOT occur during meiosis I? A) Synapsis B) The splitting of centromeres C) The pairing of homologous chromosomes D) The breaking down of the nuclear membrane

B - One of the key differences between mitosis and meiosis occurs during their respective anaphases. During anaphase of mitosis, sister chromatids are pulled apart at the centromeres, each becoming an independent chromosome in the two diploid daughter cells. During anaphase I of meiosis I, homologous pairs of chromosomes are separated into the two daughter cells. However, each chromosome still consists of two sister chromatids joined to each other at the centromere. It is not until anaphase II of meiosis II that the centromere is split and the sister chromatids separate. Thus, B is the best answer.

If there is an excess of ATP present: A) Fructose 1-6 biphosphatase will be down regulated, and will less actively convert fructose six phosphate to fructose 1-6 biphosphate. B) Phosphofructokinase one will be down regulated and will less actively phosphorylate fructose six phosphate. C) Glycolytic pathway will be up regulated. D) Gluconeogenesis pathway will be down regulated.

B - Phosphofructokinase one phosphorylates fructose six phosphate and one of the three regulated steps glycolysis. ATP, the product of glycolysis, will allosterically inhibit the reaction.

Palmitic acid has the chemical formula, C16H32O2. Which of the following best describes the appropriate product distribution after 4 rounds of beta-oxidation? A) 1 C12H24O2; 4 NADH, 4 acetyl-CoA; 4 FADH2. B) 1 C8H16O2; 4 NADH; 4 acetyl-CoA; 4 FADH2. C) 1 C8H16OSCoA; 4 NADH; 4 acetic acid; 4 FADH2. D) 1 C6H12O2; 5 NADH; 5 acetyl-CoA; 5 FADH2.

B - Recall that each round of beta-oxidation produces 1 acetyl-CoA, 1 NADH, 1 FADH2, decreasing the fatty acid chain length by two carbons. After 4 rounds of beta-oxidation, there will be 4 acetyl-CoA produced (choices C and D are wrong), 4 NADH, 4 FADH2 and a fatty acid that is reduced by 8 carbons, C8H16O2 (choice A is incorrect; choice B is correct)

Which of the following enzymes are involved in DNA replication in human cells? I. Taq polymerase II. Telomerase III. Ligase IV. Transcriptase A) I and III B) II and III C) II and IV D) I, II, and III

B - Telomerase is required for DNA replication, as it contributes to maintaining the integrity of the genome. Because DNA polymerase reads the template 3' to 5' and ca only begin replication where a primer has been established, the 3' ends of chromosomes generally are not replicated. Telomerase adds a repeating sequence to prevent these ends from shortening during every replication cycle. Ligase is also necessary, and has the role of joining the Okazaki fragments on the lagging strand; Taq has a bacterial origin and is used in PCR; Transcriptase enzymes are involved in transcription, not replication.

Myrcene is a monoterpene with a pleasant aroma found in several plants. What is the molecular formula of myrcene? A) C5H8 B) C10H16 C) C15H24 D) C20H32

B - Terpenes are a broad class of compounds composed from isoprene units (C5H8). Monoterpenes, consist of two isoprene units, resulting in a molecular formula of C10H16. Hemiterpenes (C5H8) Sesquiterpenes (C15H24) Diterpenes (C20H32)

Heart murmurs are extra, abnormal sounds (beyond the normal closure of the valves) produced during the cardiac cycle. They can be caused by stenotic (stiffened) valves, or by valves that do not close properly and allow regurgitation. Murmurs are classified as diastolic or systolic depending on when the additional sound is produced. A heart murmur caused by a failure of the AV valves to close properly would most likely be classified as a: A) Diastolic murmur, because this would allow flow from the atria to the ventricles during diastole. B) Systolic murmur, because this would allow regurgitation of blood from the ventricles to the atria during systole. C) Diastolic murmur, because this would allow regurgitation of blood from the arteries to the ventricles during diastole. D) Dystolic murmur, because this would allow additional blood to flow from the atria to the ventricles during systole.

B - The AV valves close at the beginning of systole to prevent regurgitation of blood into the atria while the ventricles are contracting. If the AV valves failed to close properly, blood from the high-pressure ventricles would flow back into the low-pressure atria during systole and would produce an abnormal murmur. Flow from the atria to the ventricles during systole would be prevented by the pressure gradient (choice D is wrong). Blood normally flows from the atria to the ventricles during diastole, through the open AV valves; this would not produce a murmur (choice A is wrong), and these valves do not separate the ventricles and the arteries so this would not affect blood flow between those regions (choice C is wrong).

While the blood is buffered primarily through the equilibrium between carbon dioxide and carbonic acid, coupled with hemoglobin, the blood may also be buffered through other plasma proteins. Which of the following is true? A) A shift in the pH can alter the tertiary or quaternary structure of the protein, allowing it to buffer the pH by precipitating out of plasma in response to pH shifts. B) The amino acid residues that make up the protein may act as Brønsted acids or bases, reducing shifts in pH. C) Plasma soluble proteins have enzymatic function allowing them to sequester hydronium ions from the blood inside membrane-bound organelles in the podocytes lining the capillaries. D) In the presence of altered pH, any plasma-soluble proteins will undergo either acid- or base-catalyzed cleavage, thus depleting the acid or base causing the disruption to blood pH.

B - The amino acids that make up a protein may include many acidic or basic side chain groups. Those side chains can either release or absorb protons, allowing them to help buffer the blood through action as a Brønsted-Lowry acid or base.

A patient receiving a liver transplant has immediate post-operative complications, and subsequent blood tests and biopsy results indicate organ failure. Which type of immunity is most likely involved in this process? A) Innate B) Humoral C) Cell-mediated D) Complement system

B - The clinical presentation of this patient is most consistent with "hyperacute" rejection, given that the organ began to fail in the immediate post-operative period. Hyperacute rejection is described in the passage as being attributed to antibodies. Antibodies are secreted by B-cells into the blood and are classified as humoral immunity. In contrast, acute rejection (days to weeks) is said to be due to a T-cell mediated response, which is classified as cell-mediated immunity (choice C is wrong). Innate immunity refers to the general, non-specific protection the body provides (e.g., tears, skin, complement system) and is not involved in acute rejection (choices A and D are wrong).

Rank the following metals in order of increasing oxidation potential: I. Sr(s) II. Cs(s) III. Fe(s) IV. Ca(s) A) III < II < I < IV B) III < IV < I < II C) II < I < IV < III D) IV < I < III < II

B - The greater an element's oxidation potential, the more readily the atom will lose an electron and the more reactive the element is. This can be approximated by the relative ionization energies of the elements. In general, ionization energies increase across a row and decrease down a column of the periodic table. Elements farthest to the left are most reactive, and reactivity often increases down a metal family, especially for groups I and II. Since this is a ranking question, look for an extreme and eliminate answer choices. Cs is at the bottom of the alkali metal family, so should therefore have the largest oxidation potential. Eliminate choices A and C. Looking at the remaining choices, next determine whether Fe or Ca is more likely to lose electrons. Since transition metals are less reactive than alkaline earth metals eliminate choice D.

Sodium laurel sulfate, a common ingredient in hand soap, functions due to the combination of its hydrophilic head (pKa = 1.9) and hydrophobic tail. This amphipathic nature allows for the removal of hydrophobic substances with water. Which of the following would most likely increase the effectiveness of sodium laurel sulfate? A) Benzene B) Dilute aqueous sodium bicarbonate C) Aqueous phosphoric acid (pH = 1.9) D) Lemon juice

B - The head of sodium laurel sulfate (SLS) is a weak acid. It is most hydrophilic when deprotonated and charged. This head can be protonated in lower pH solutions and become less effective. Therefore, addition of a hydrophilic weak base like sodium bicarbonate (NaHCO3) would favor the deprotonated, active form of SLS. Choices C and D are both acidic and eliminated. Benzene is a hydrophobic compound with essentially non-acidic protons (pKa of about 43), eliminating choice A.

If analysis of the patient's semen revealed decreased levels of fructose, which structural components of mature sperm would be directly affected? I. Head II. Midpiece III. Tail A) I only B) II only C) I and II only D) I, II, and III

B - The midpiece of the sperm contains mitochondria, which generate ATP from fructose. This energy is used to propel the sperm through the female reproductive tract by utilizing a flagellum. A: The head of the sperm contains the genetic material and is also covered by an acrosome, which is necessary to penetrate the ovum. D: The tail of the sperm would not be directly affected by decreased levels of fructose. However, it could arguably be indirectly affected, due to less ATP generated by the mitochondria located in the midpiece.

If oligonucleotides such as mRNA were not degraded rapidly by intracellular agents, which of the following processes would be most affected? A) The production of tRNA in the nucleus B) The coordination of cell differentiation during development C) The diffusion of respiratory gases across the cell membrane D) The replication of DNA in the nucleus

B - The question asks the examinee to identify the process most likely to be affected if oligonucleotides, such as mRNA, were not degraded rapidly by intracellular agents. The destruction of mRNA prevents continuous protein production, allowing the cell to change its protein expression over time. B is the best answer because the coordination of cell differentiation during development is extremely sensitive to the timing of mRNA turnover. A and D are not the best answers because the exact timing of mRNA turnover is less critical to the successful completion of tRNA production (A) and DNA replication (D). C is incorrect because it is unlikely that an accumulation of mRNA would affect the diffusion of respiratory gases across the cell membrane.

The outer layers of human skin are composed of dead cells impregnated with keratin and oil, which make the epidermis relatively impermeable to water, yet humans sweat freely in hot temperatures. This occurs because: A) The salt in sweat allows it to diffuse through the skin. B) Sweat glands have special channels through the skin. C) An osmotic gradient in sweat moves it through the skin. D) Sweating occurs in only those areas of the body where the skin is water permeable.

B - The sweat glands secrete onto the surface of the skin through channels continuous with the most superficial layer of the skin, the epidermis. These channels prevent water loss by isolating the water-permeable, sweat-secreting cells from dry surface air. The openings of the sweat glands on to the surface of the epidermis are pores. The correct answer is B. All the other answers require some movement of water through the epidermis itself, which is relatively impermeable.

The urea cycle is a cyclic process similar to the citric acid cycle. Ornithine is regenerated during one of the cycle and use for the next. Which pairing correctly matches the reactants and products of the cycle, respectively? A) Water in ammonium, urea. B) Carbon dioxide in ammonia, urea. C) Urea in water, uric acid. D) Oxygen and ammonia, urea.

B - The urea cycle produces urea from ammonia, it is necessary to prevent the buildup of ammonia, a toxic nitrogenous product. The process is cyclic, and the first set of reactants include ATP, ammonia, and carbonate, which is equivalent to water and carbon dioxide. The end products of the cycle are urea in ornithine, the latter of which is reused during the next series of reactions. C: While uric acid is present in the body, it is not the end product of the urea cycle. Buildup of uric acid can lead to get out and is associated with diabetes and kidney stones.

The urinary system is most closely tied to which of the other body systems? A) The integumentary system. B) The endocrine system. C) The digestive system. D) The skeletal system.

B - The urinary system includes the kidneys as well as organs you specifically for excretion (a urinary bladder, ureter, and urethra). This system is heavily dependent on hormonal input from the endocrine system, especially with regard to water balance. A,C,D: The integumentary system (skin), digestive system, and skeletal system are all dependent on the kidneys for proper functioning. However, none of them display the same close relationship with the excretory system as choice B.

Thin-film interference is observed when polychromatic light is incident on an interface formed when one semitransparent medium is layered on top of a second semitransparent medium. Which of the following best explains why thin-film interference generates a multicolored array when two semitransparent fluids are used? A) The density of the top fluid varies along the fluid interface. B) The thickness of the top fluid varies along the fluid interface. C) The osmolarity of the top fluid varies along the fluid interface. D) The two semitransparent fluids decrease the number of reflection events at the fluid interface.

B - Thin-film interference refers to the multicolored arrays generated by the reflection events that occur within a system composed of two layers of semitransparent media. The relative intensity of the colors seen within the multicolored array depends on the constructive and destructive interference among waveforms of polychromatic (variable wavelength) light. Interference that occurs within a thin-film system results from differences in the travel path of the light. Some light that is incident on the two-media system will reflect off the surface of the first medium (thin film), and some light will transmit through the first medium before reflecting off the media interface. When light reflected off the surface of the first medium constructively interferes with light reflected off the media interface, an observer perceives bright light. Conversely, when light reflects off the surface of the first medium and destructively interferes with light reflected off the media interface, an observer perceives darkness. Thin-film interference is influenced by the thickness of the film because light waves of a specific wavelength (ie, color) that interfere constructively or destructively at one film thickness may no longer interfere in the same fashion at a different thickness. Therefore, surface tension and other effects that locally distort the surface of a fluid may also cause multicolored arrays to appear on the surface as light that emerges from adjacent segments of the thin film varies in color composition.

In miRNA-directed gene silencing, a small RNA binds to an mRNA and directs degradation of the mRNA or prevents translation of the mRNA. Which of the following terms describes the process through which binding occurs? A) RNA polymerization B) Hybridization C) Elongation D) Transcription

B - This is a question of definition; only hybridization describes a process of binding through complementary nucleotides.

The Goosecoid protein (transcription factor) would contain which of the following? A) Nuclear localization signal and disulfide bonds B) Nuclear localization signal and DNA binding motifs C) Signal sequence and disulfide bonds D) Signal sequence and DNA binding motifs

B - This is a two-by-two question, where two decisions need to be made to find the correct answer. Since Goosecoid is a transcription factor, it must be localized in the nucleus. Nuclear proteins contain a nuclear localization signal, not a signal sequence, which targets proteins to the secretory pathway (choices C and D can be eliminated). Since it is a transcription factor, Goosecoid binds DNA and must therefore have DNA binding motifs (choice B is correct and choice A is wrong).

Which of the following is an accurate statement regarding mitosis and meiosis? A.Mitosis results in two haploid daughter cells. B.During mitosis and meiosis II, sister chromatids are separated. C. During meiosis I and meiosis II, sister chromatids are separated. D. Meiosis I results in two diploid daughter cells.

B - This question is asking us to recall some facts about mitosis and meiosis. Remember that mitosis separates sister chromatids to create two diploid daughter cells. Meiosis I separates homologous chromosomes to create haploid daughter cells, each of which divides again, separating sister chromatids in Meiosis II to create two haploid cells. Meiosis I results in two haploid daughter cells.

One of the fundamental qualities of DNA polymerase is its inability to initiate replication. In humans, it can only begin to extend a strand when a short RNA primer is already present. A scientist thinks he has discovered a retrovirus that employs tRNA to initiate DNA synthesis rather than traditional primers. Is this possible? A) Yes, tRNA has an innate ability to mimic the function of enzymes such as primase B) Yes, both of the tRNA molecule and the RNA primers provide a free 3 OH group as an attachment site for enzymes that elongate the chain C) No, retroviruses utilize reverse transcriptase instead of DNA polymerase D) No, tRNA only functions in the assembly of proteins at viral ribosomes

B - This situation is possible, and describes how retroviruses generally synthesize DNA. Eukaryotic DNA polymerfase requires a free 3' OH group, which is provided by a short RNA strand synthesized by primase enzymes. Retroviruses, use reverse transcriptase to elongate the 3' OH provided by the tRNA. A) tRNA does not mimic any enzymatic functions C) This is true but reverse transcriptase has a DNA polymerase activity and initiates synthesis by binding to a specific tRNA D) Viruses do not contain ribosomes or other organelles

A typical eosinophil would be expected to differ from an erythrocyte in that the eosinophil: A) Would not contain a membrane-bound nucleus. B) Would contain a membrane-bound nucleus. C) Could not be expected to be found in human blood. D) Would be expected to be found in human blood.

B - Unlike almost all cells in the human body, erythrocytes (red blood cells) do not contain DNA or a membrane-bound nucleus. This allows these cells to have more space for oxygen-carrying hemoglobin molecules. Thus, eosinophils differ from erythrocytes in that eosinophils do contain nuclei and DNA.

A cardiologist takes a biopsy of muscle tissue from the wall of the superior vena cava, Upon analysis of his tissue, what characteristics would he find? A) Multinucleate, striated fibers with clearly defined sacromere B) Uninucleate, non-striated cells with tapered ends C) Highly branched, striated fibers that mainly appear to be uninucleate D) As a vein, the vena cava does not contain a muscle

B - Veins and arteries, arterioles, and venues contain a layer of smooth muscle in their walls, allowing for vasoconstriction/dilation. Smooth muscle are uninucleate, rounded cells with tapered ends, and a lack of striation

A method for increasing the quantity of aquaporins in the collecting duct cells of the kidney during a certain period would be: A) Increasing extracellular sodium ion concentrations B) Stimulating aldosterone production. C) Stimulating antidiuretic hormone production. D) Increasing blood pressure

C - An increase in aquaporins in the collecting duct cells of the kidneys would increase the reabsorption of water in the nephrons, causing more water to remain in the body rather than in the urine. For example, if blood pressure were low, an increase in aquaporins would increase the amount of water reabsorbed into the blood. This would increase the blood volume and, in turn, the blood pressure. Conversely, if blood pressure were high, having fewer aquaporins in collecting duct cells would decrease the amount of water reabsorbed into the blood. This would decrease blood volume and thus decrease blood pressure, maintaining homeostasis (Choice D). The hormone responsible for activating a pathway that leads to the insertion of more aquaporins in collecting duct cells is antidiuretic hormone (ADH) or vasopressin. By stimulating ADH production, aquaporin insertion would increase in collecting duct cells, causing an increase in water retained by the body. (Choice A) An increase in extracellular sodium ion concentrations would increase the osmolarity (solute concentration) of the extracellular fluid, causing more water to remain in the body (ie, via osmosis) to maintain homeostasis. This would increase blood pressure, which would in turn inhibit ADH production, causing a decrease in the quantity of aquaporins in kidney collecting duct cells. (Choice B) Aldosterone is a hormone stimulated by the renin-angiotensin system that acts on the distal tubules and collecting ducts of nephrons to promote the reabsorption of sodium ions and the secretion of potassium ions. Increased reabsorption of sodium ions increases the osmolarity (solute concentration) of the renal interstitial fluid. This elevated osmolarity promotes water reabsorption, which ultimately causes blood volume and blood pressure to increase. An increase in blood volume would inhibit the production of ADH, leading to a decrease in aquaporins in collecting duct cells.

With which of the following do the arginine side chains found on histone proteins most likely interact? A) Thymine groups on DNA B) Adenine groups on DNA C) Phosphate groups on DNA D) Oxyribose on RNA

C - Arginine is a basic amino acid, meaning that it is positively charged at physiological pH. Positive species interact best with negative species, and of the answers listed, only phosphate groups are negatively charged.

Centrifugation of blood with an anticoagulant agent separates the blood components into layers by density. The least dense layer is the plasma. The plasma layer is most likely to include which component? A) Platelets B) Albumin C) Hemoglobin D) Neutrophils

C - Blood is the only fluid connective tissue in the body and is composed of living blood cells and nonliving plasma. Centrifugation with an anticoagulant agent separates these components by density, with plasma as the least dense layer. Plasma is the liquid portion of blood and is ~90% water, with the remaining 10% composed of other substances (ie, water, electrolytes, gases, hormones, nutrients, metabolic waste). One substance found in plasma is blood proteins, such as albumin. Because albumin cannot easily cross capillary pores due to its size, it is an important solute for maintaining capillary oncotic pressure (ie, pressure resulting from solute concentration differences between the capillaries and interstitial fluid). The oncotic pressure maintained by solutes such as albumin causes a pulling force for water entry into capillaries via osmosis. This pressure is balanced by the pushing force of blood against capillary walls (hydrostatic pressure), which helps keep fluid in the vasculature. The substances in the other answer choices would be found in the cellular components of the blood, which originate in the bone marrow: Platelets are cell fragments important in the clotting of blood (Choice A). Hemoglobin is a protein that binds respiratory gases found within erythrocytes (ie, red blood cells) (Choice C). Neutrophils are a type of leukocyte, or cells involved in the immune defense (Choice D).

During normal development in the seminiferous tubules, what class of Tex11 non-expressing somatic cells is responsible for nourishing sperm cells? A) Leydig cells B) Chromaffin cells C) Sertoli cells D) Granulosa cells

C - During spermatogenesis, the main function of Sertoli cells is to nourish the developing sperm cells. These cells are located in the epithelial lining of the seminiferous tubules and are activated by FSH. A: Leydig cells are adjacent to the seminiferous tubules in the testicles. In response to stimulation by LH, they produce testosterone and other androgens. B: Chromaffin cells are catecholamine-secreting neuroendocrine cells of the adrenal medulla. Remember, the catecholamines include epinephrine and norepinephrine. D: Granulosa cells are follicular cells closely associated with the developing female oocyte (egg). They function to convert thecal androgens to estradiol prior to ovulation. After ovulation, they give rise to the corpus luteum and begin producing high levels of progesterone.

Basic amino acids are often found in the active site of enzymes. Which of the following is not a possible explanation of this observation? A) These amino acids can covalently bind to substrates. B) These amino acids can hydrogen bond with a substrate to keep it in place. C) These amino acids can alter the pH of the mini environment. D) These amino acids can take part in the enzymatic reaction.

C - Enzymes work best at physiological pH which differ significantly from the PK values of the side chains of basic enzymes. The presence of such residues in the active site therefore do not affect the pH of the environment surrounding the site itself. A: The side chain amino group can be de protonated by the substrate, then covalently attached to the substrate using its new lone pair of electrons. B: The basic side chain is usually positively charged, allowing it to autism electrophilic center. It cannot attract and interact with nucleophilic substrates. D: In many cases, the amino acid itself does participate in the enzymatic reactions, reacting with a substrate in an acid base or a nucleophile electrophile fashion.

Epithelial cells connect to underlying layers of connective tissue through cell adhesion proteins such as: A) Interleukins B) Cadherins C) Integrins D) Connexins

C - Epithelial cells do not directly connect to collagen and other fibers of the basement membrane. instead they possess transmembrane proteins called interns that can connect to collagen, to a bridging protein like fibronectin, which itself attaches to collagen a- interleukins are immune proteins that are not involved in cell ECM junctions b- cadherins form cell cell junctions, not junctions between the cell and ECM d-connexins are gap junction proteins and are not involved in this type of connection

In comparison to a neural cell, a gastrointestinal cell would likely spend: A. Less time in S phase. B. Less time in M phase. C. Less time in G0. D. The same amount of time in all cell cycle phases.

C - G0 is a non-growing state that accounts for the observed differences in length in the cell cycle. While intestinal cells will divide twice per day, neuronal cells do not divide following initial differentiation and therefore remain permanently in G0. A: Since gastrointenstinal cells divide more often than neurons, they would be expected to spend more time in S (DNA synthesis) phase. B: Since gastrointenstinal cells divide more often than neurons, they would be expected to spend more time in M (mitotic) phase. D: The same amount of time in all cell cycle phases. Since gastrointenstinal cells divide more often than neurons, they would be not be expected to spend the same amount of time in all cell cycle phases.

Which histone protein is associated with the sealing off of the DNA as it enters and exits the nucleosome? A) H2A B) H2B C) H1 D) H3

C - H1 is the component of the nucleosome that is positioned outside the main histone bead structure; this protein holds the DNA in place as it enters and exits the wound region of the nucleosome; also provides stability to the structure

Phosphofructokinase (PFK) is the rate-limiting enzyme in glycolysis. Which of the following conditions would negatively regulate PFK activity? A) Low cellular ATP/ADP B) Low cellular NADH/NAD+ C) High citric acid levels (Krebs cycle) D) High cellular Mg2+ levels (a cofactor for the PFK reaction)

C - High citric acid levels suggest increased substrate supply (acetyl-CoA) to the Krebs cycle (citric acid cycle). This would negatively regulate PFK in order to slow down glycolysis, thus reducing the availability of acetyl-CoA to the Krebs cycle. Low ATP/ADP and NADH/NAD+ ratios suggest that the energy levels of the cell are low and therefore would activate PFK (choices A and B are wrong). Since Mg2+ is a cofactor for the PFK reaction, its presence is necessary. Thus, low levels of Mg2+ would reduce the rate of the reaction, but high levels would not affect it in any way (choice D is wrong).

A woman suffers a hemorrhage into her pituitary gland, destroying only the portion of the anterior pituitary responsible for producing luteinizing hormone. What will be the effect of this destruction on her menstrual cycle? A) Her menstrual cycle will not be affected by the destruction. B) She will still ovulate and release estrogen, but not progesterone. C) She will become anovulatory. D) She will still ovulate and release progesterone, but not estrogen.

C - If the portion of the anterior pituitary that produces LH is destroyed, then LH levels will not surge during the menstrual cycle and ovulation would not occur (choice C is correct and choices B and D are wrong). In the absence of ovulation, the corpus luteum would not develop either, although if FSH is unaffected, the follicle would most likely develop and release estrogen. Regardless, the menstrual cycle would be affected (choice A is wrong).

Which of these choices correctly identifies the function of a specific antibody? I. IgA - present in the mucosal areas to prevent colonization by pathogens. II. IgE - binds to allergens, causing the release of histamine. III. IgM - part of the early humoral response, secreted as a pentamer. IV. IgD - responsible for the majority of the humoral response. A) I only B) I and III C) I, II, and III D) I, III, and IV

C - IgA is an antibody found in the mucous membranes that help prevent initial colonization by pathogens. IgE is primarily involved in the response to allergens, while IgM is responsible for the initial humoral response. IgG is the antigen receptor for bee cells that have not yet been exposed to antigens.

Yeast cells can grow under either aerobic or anaerobic conditions. If the same concentration of glucose were used to grow two different yeast colonies, would the growth rate be faster under aerobic or anaerobic conditions? A) Anaerobic, because the final product (alcohol) would contain more energy than the final product of respiration (H2O) B) Anaerobic, because the cells would not have to produce the enzymes needed for the citric acid cycle C) Aerobic, because a much greater amount of ATP would be produced to provide energy for reproduction D) Aerobic, because the CO2 produced in fermentation would be toxic to the culture

C - In aerobic conditions, the cells produce 30 molecules of ATP, whereas they only produce 2 in anaerobic conditions. The organisms can use the ATP to power their growth. A: In anaerobic conditions, the organism is only able to produce 1 molecules of ATP, thus the organism will not have much energy to use to grow in anaerobic conditions. It is true that alcohol has more energy than water, but because this energy is not free and cannot be converted in ATP, the organism cannot use it to grow. B: Although it is true that synthesis of enzymes for the citric acid cycle requires energy, anaerobic respiration results in the production of only 2 molecules of ATP and this value is much lower than the 30 molecules produced with aerobic respiration. Thus, an organism that uses only anaerobic respiration will grow less than an organism that uses aerobic respiration because it will have a lower level of ATP available. D: Although in aerobic conditions the cells produce more ATP, it is not true that the CO2 produced is toxic to the cells. Cells have evolved to survive the levels of CO2 that are produced during fermentation.

A bacterium has a faulty lac operon in which there is a structural defect in the operator. In this bacterium: A) There is a mutation in a segment of DNA that binds a promoter. B) A missense mutation is found in the gene that codes for the repressor. C) There is a structural problem with a segment of DNA that binds a repressor. D) There will be no proteins available capable of digesting lactose.

C - In the lac operon, the operator is the segment of DNA that binds to the repressor. In the absence of functional repressor/operator binding, the cell will constitutively produce the proteins needed for lactose metabolism. Although this may not be fatal to the cell, it will waste energy if the surroundings lack lactose. A: The promoter is a segment of DNA. The promoter binds to the RNA polymerase, not the promoter. B: The operator binds to the repressor, not codes for the repressor. The repressor is coded elsewhere. D: This is false; without functional operator/repressor binding, the cell's lactose-digesting machinery will be "stuck on."

Diabetes mellitus is characterized by dysregulation of the hormone insulin. Which of the following statements about insulin is/are correct? I. Insulin is produced by the beta cells of the pancreas. II. Insulin directly acts on glucose in the blood. III. Insulin is a peptide hormone. A) I only B) I and II only C) I and III only D) I, II, and III

C - Insulin is produced by the beta cells of the pancreas (I). Insulin is a peptide hormone, as is characteristic for hormones that have a quick-acting effect (and that end in -in) (III). D: Insulin does not directly interact with glucose in the bloodstream; instead, it acts to reduce blood glucose levels by promoting the uptake of glucose into cells (II).

In an adult, which of the following cell types is LEAST likely to enter a programmed G0 phase of the cell cycle? A. Liver cells B. Kidney cells C. Epithelial cells D. Neurons

C - Liver cells are more likely to be found in G0 because they do not divide as often as epithelial cells. B: Kidney cells are more likely to be found in G0 because they do not divide as often as epithelial cells. C: For this question, we need to know what the G0 phase (shown below along with the rest of the cell cycle) entails. This is the state that a cell will enter if it does not need to divide. Since epithelial cells are those that divide the most out of the options, choice C is correct. D: Neurons in adults do not divide and are almost always found in G0.

Post transcriptional silencing is theorized to prevent some gene expression in response to stress. Which of these modifications could prevent functional mRNA from being translated? A) Methylation B) Acetylation C) Binding to miRNA D) Binding to tRNA

C - MicroRNAs are responsible for post-transcriptional silencing, these short ncRNAs molecules bind to mRNA, signaling its destruction or promoting degradation of its protective post-transcriptional modifications A, B: Both methylation and acetylation are forms of pre-transcriptional silencing D: tRNA mainly plays a role in the proper translation of processed mRNA

Restriction enzymes recognize palindromic sequences in intact double stranded DNA helices. Since the bases are located on the inside of the strand surrounded by the sugar phosphate backbone, how does the enzyme accomplish this? A) Restriction enzymes cleave only when helicase has unwound DNA double helix during replication. B) Restriction enzymes bind to the minor groove is the double helix, allowing them to find the recognition sites by hydrogen bonding with the palindromic bases. C) Restriction enzymes bind to the major groove in the double helix, allowing them to find the recognition sites by hydrogen bonding with the palindromic bases. D) Restriction enzymes bind to the backbone of the double helix, allowing them to find the recognition sites by hydrogen bonding with phosphate groups corresponding to the palindromic basis.

C - Most proteins that interact with the intact a double helix bind to the major groove. This allows unambiguous interactions between the amino acids of the protein and the DNA bases via H bonding. a: Restriction enzymes do not require DNA strands to have already been on wound. b: The minor groove does not allow for base specific unambiguous interactions. c:Proteins may bind the phosphates in the DNA backbone, but the phosphate groups are identical regardless of whether the associated base is A, C, T or G. In other words, this would not allow for sequencing specific interaction.

Women over 35 years of age have an increased risk of nondisjunction due to errors in what phase of meiosis? A) Prophase I B) Metaphase I C) Anaphase II D) Prophase II

C - Nondisjunction is the failure of chromosomes to separate properly during anaphase I of meiosis or the failure of sister chromatids to separate properly during anaphase II of meiosis. A, B, D: Nondisjunction will not occur in this phase.

Which of the following are DNA-binding proteins that are NOT involved in eukaryotic transcription or transcriptional regulation? A) Transcription factors B) RNA polymerase C) Nucleaases D) Histones

C - Nucleases cleave DNA molecules and are not involved in transcription; a) transcription factors bind to specific DNA sequences to regulate the rate of transcription b) rna polymerase synthesize mRNA molecules during transcription d) histones pack nucleic acids into chromosomes and regulates transcriptional activity

Drinking ocean water is ultimately fatal to a human because: A) The water has a very low solute concentration relative to the body's cells, resulting in cell swelling and death. B) The kidney must work very hard to excrete the excessive levels of bivalent ions in the ocean water, causing kidney failure. C) The water has a very high solute concentration relative to the body's cells, resulting in cell shrinkage and death. D) Ocean water contains toxic levels of environmental pollutants that can damage organs or cause cancer, leading to death.

C - Ocean water contains very high levels of sodium and chloride ions. This creates an osmotic gradient that pulls water out of the body's cells, leading to dehydration. A schematic of a cell in such a hypertonic (solute-rich) environment is shown below.

When used as described in the passage, antisense drugs (bind to antisense mRNA) prevent: A) DNA replication. B) RNA transcription. C) RNA translation. D) Cell replication

C - RNA is directly involved in translation. However, because RNA can hybridize with either RNA or DNA, a role for antisense RNA in the regulation of DNA replication, transcription, and even cell replication may be possible, but these processes are not the focus of the passage.

The concentration of intracellular signaling molecules fluctuates rapidly in dividing cells during the cell cycle. Which of the following experimental techniques would be best to elucidate the mechanism of regulation for these proteins? A. RT-PCR and Southern blot B. Southern blot and northern blot C. Western blot and RT-PCR D. Western blot and Southern blot

C - Rapidly dividing cells undergo mitosis under the influence of specific signaling molecules. These molecules are expressed when their genes are transcribed, then are translated into proteins. In order to gain the best understanding of how a signaling protein's levels are regulated, both the protein and mRNA levels would need to be studied. Western blotting gives us information about the amount of protein expressed in a cell, while RT-PCR gives us information about the amount of RNA expressed.

Those species that are capable of both sexual and asexual reproduction will typically prefer sexual reproduction because it: A) Increases the likelihood of each individual offspring surviving. B) Increases the likelihood of beneficial mutations. C) Creates more variation in the next generation. D) Takes less time to complete.

C - Sexual reproduction involves "shuffling the genetic deck" when recombining the genes of two parents. This significantly increases variation in the next generation. This increase in variability helps improve the survival of the whole species by allowing it to adapt more quickly to changing selection pressures.

The individual symptom of primary isolated dystonia listed in the passage results in shortening of which of the following regions within skeletal muscle? I. A-band II. I-band III. H-zone IV. Z-line A) I only B) I and II only C) II and III only D) I, II, III, and IV

C - The I-band is the region exclusively containing thin, actin filaments. The H-zone is the region exclusively containing thick, myosin filaments. Both of these regions shorten in response to skeletal muscle contraction. B: The length of the A-band remains unchanged during skeletal muscle contraction. D: The Z-line defines the boundaries of each sarcomere unit. It does not lengthen or shorten in response to skeletal muscle contraction.

Some of the DNA sequences that are eliminated during macronuclear differentiation (Figure 1, Step 6) may be sequences involved in: A) Transcription. B) Translation. C) Meiosis. D) Ribosome production

C - The macronucleus does not go through the process of meiosis. Genes involved in meiosis are therefore superfluous in this genome. The correct answer is choice C, that DNA sequences involved in meiosis may be eliminated. On the other hand the primary use of the macronucleus is to provide the proteins for the cell's day-to-day functioning. The functions of transcription (choice A), translation (choice B) and ribosome production (choice D) must be coded for by macronuclear genes because they are necessary for it to direct protein synthesis.

In almost all vertebrates, when the optic cup fails to develop in the embryo, the lens also fails to form. This constitutes evidence that: A) The process of neurulation follows gastrulation. B) The eye develops early in vertebrate morphogenesis. C) Cells may induce neighboring cells to differentiate. D) Cell differentiation is an "all or none" phenomenon.

C - The optic cup develops from a bulge on the side of the developing brain, which influences the overlying ectoderm to produce the lens. It is therefore an example of cells inducing neighboring cells to differentiate, so option C is the correct answer. The other response choices are irrelevant to the question, so they are not good answers. The absence of the optic cup and lens has no influence on timing of neurulation relative to gastrulation (choice A). The presence or absence of the optic cup and lens has no effect on the timing of eye development (choice B). Cell differentiation is not an "all or none" phenomenon

Which sequence properly indicates the transmission path of impulses as a direct result of transcranial magnetic stimulation? A) Sensory neurons, cerebral cortex, afferent neurons, skeletal muscle cells B) Skeletal muscle cells, afferent neurons, spinal cord, cerebral cortex C) Cerebral cortex, spinal cord, efferent neurons, skeletal muscle cells D) Efferent neurons, afferent neurons, interneurons, cerebral cortex

C - The question is basically asking for the sequence in which information travels from the CNS to the periphery. This sequence is cerebral cortex → spinal cord → efferent neurons → interneurons → motor neurons → muscle tissue.

To be an effective therapy, an antisense gene that is incorporated into a genome that contains the target gene must be: A) On the same chromosome as the target gene but not necessarily be physically adjacent. B) On the same chromosome as the target gene and must be physically adjacent. C) Regulated in a similar manner as the target gene. D) Coded on the same strand of DNA as the target gene

C - To provide effective therapy, this antisense gene would need to be regulated in a manner similar to the manner in which the target gene is regulated so that the antisense RNA is produced at the same time that the sense mRNA is produced. This would ensure that the antisense RNA is available to bind with the sense mRNA, thereby preventing its subsequent translation. As a result, C is correct. A, B, and D are incorrect because the key feature determining whether an antisense drug will work is the timing of the expression of the antisense gene. This is controlled by specific regulatory elements, not necessarily the location of the antisense gene relative to the target gene.

Which of the following molecules would be expected to have the lowest tissue concentrations in active skeletal muscle deprived of O2? A) Glyceraldehyde 3-phosphate B) Lactate C) Citrate D) Pyruvate

C - Under anaerobic conditions (in the absence of oxygen), pyruvate undergoes fermentation to lactate in the cytoplasm instead of being transported to the mitochondria for conversion to acetyl-CoA. Thus, acetyl-CoA will not be present to enter the Krebs cycle and will not be converted to citrate. A: This is a glycolytic intermediate. Glycolysis does not require oxygen and proceeds similarly under aerobic and anaerobic conditions. B: As a product of anaerobic fermentation, lactate will likely display increased concentrations in the absence of oxygen. D: Pyruvate is formed during glycolysis, which continues regardless of the presence of O2. While pyruvate is later converted to lactate via fermentation, there is no reason to suspect that this would lower its concentrations any more than its conversion to acetyl-CoA under aerobic conditions

DNP is a toxin that describes a proton gradient created by the electron transport chain. Where in the cell does DNP act? A) Plasma membrane. B) Mitochondrial matrix. C) Inner mitochondrial membrane. D) Nuclear membrane.

C - the electron transport chain is present on the inner mitochondrial membrane and pumps protons from the matrix into the inter-membrane space. DNP is an uncoupling agent which allows protons to leak back across membrane, therefore destroying the gradient.

Albuterol is a sympathomimetic drug that acts on smooth muscle. This medication is frequently packaged in rescue inhalers used by asthmatic patients to relieve acute asthma and enable a return to normal breathing. Of the following cells, which are most likely to display receptors for albuterol? A) Smooth muscle cells surrounding alveolar sacs B) Smooth muscle cells surrounding the trachea C) Smooth muscle cells surrounding the terminal brinchioles D) Smooth muscle cells surrounding the glottis

C -Asthmatic patients suffer from uncontrollable constriction of the bronchioles, a condition that results in the characterstic wheezing with an asthma attack; although the sympathetic nervous system normally triggers the contraction of smooth muscle the beta adrenergic receptors expressed bt muscle in the bronchioles promote relaxation and subsequent bronchial dilation a-the alveoli are not surrounded by muscle, they are pulled open during inspiration by a decrease in pressure in the pleural space during passive expiration, the shrink elastically b- the trachea is a rigid tube held open by catilaginous rings and is not surrounded by muscle; an upper airway obstruction is usually the result of laryngospasm which occludes the tracheal opening bu does not actually collapse the trachea d - the glottis is not closed by smooth muscle instead it is occluded when the epiglottis falls to rest on top of it

A scientist examining action potentials in mature muscle cells identifies certain muscle cells that, upon receiving an action potential, can directly transfer ions to adjacent cells. These cells would most likely include: I. Smooth muscle cells. II. Cardiac muscle cells. III. Skeletal muscle cells. A) I only B) II only C) I and II only D) II and III only

C- Myocytes (ie, muscle cells) generate forces that allow various functions to be performed in the body (eg, movement of limbs, pumping of blood, movement of food through the digestive tract). The three main types of myocytes are skeletal muscle cells, cardiac muscle cells, and smooth muscle cells, each of which exhibits specific characteristics: Skeletal muscle cells are long, cylindrical fibers that each possess many nuclei. Skeletal muscle cells contain sarcomeres and are therefore striated (ie, striped) in appearance. In addition, mature skeletal muscle cells lackgap junctions and are therefore electrically isolated from one another (ie, cannot directly pass ions/action potentials between cells). Cardiac muscle cells are short and branched cells with one or two nuclei each. These cells also possess sarcomeres that cause the cells to exhibit a striated appearance. Cardiac muscle cells are connected to adjacent cells via intercalated discs, which contain desmosomes (providing strong adhesion between cells) and gap junctions, openings between adjacent cells that permit the passage of certain molecules (eg, ions) between cells. Smooth muscle cells are short cells that are wide in the middle and tapered (ie, narrower) at each end. Each smooth muscle cell possesses a single nucleus. These cells lack sarcomeres and are therefore not striated in appearance. In addition, many types of smooth muscle cells possess gap junctions. In this question, a scientist identifies certain mature muscle cells that, upon receiving an action potential, can directly transfer ions to adjacent cells. Ions cannot permeate the lipid bilayer of cell membranes directly and therefore require openings (such as gap junctions) to pass between cells. Accordingly, these cells likely include both smooth and cardiac muscle cells, which are the only muscle cell types that possess gap junctions (Numbers I and II). (Number III) Mature skeletal muscle cells, which lack gap junctions, would not exhibit direct ion transfer between adjacent cells upon receipt of an action potential.

Increased vasoconstriction has an important role in which of the following situations? A) Causing the decrease in blood pressure associated with fainting B) Increasing blood flow to muscle during exercise C) Increasing blood flow to skin during blushing D) Maintaining blood pressure during a hemorrhage

D - As blood is lost from the circulation, reduction in vessel size helps maintain the necessary pressure to keep the blood circulating to all body tissues. Vasoconstriction, the narrowing of a vessel, restricts blood flow to an organ and can increase blood pressure, whereas vasodilation has the opposite effect. Increased vasoconstriction is important in maintaining blood pressure during a hemorrhage. Vasodilation increases blood flow to both the muscle during exercise and the skin during blushing.

An intravenous infusion causes a sharp rise in the serum level of albumin (the major osmoregulatory protein in the blood). This will most likely cause an: A) Increase in the immune response. B) Increase in tissue albumin levels. C) Outflow of blood fluid to the tissues. D) Influx of tissue fluid to the bloodstream

D - Because albumin has nothing to do with the immune response, answer choice A is incorrect. The plasma proteins can not cross the walls of blood vessels, but water molecules can. The wall of the artery acts as a semipermeable membrane setting up the conditions needed for osmosis to occur. An increase in plasma albumin will upset the osmotic balance because the blood will become hypertonic with respect to the tissue. Water will have to flow into the bloodstream to reestablish equilibrium. One of the causes of edema, increased fluid in body tissues, is a decrease in the plasma protein level. This occurs, for instance, in starvation when the body is forced to use its albumin as an energy source. An increase in the plasma protein level would have the opposite effect: fluid would enter the bloodstream (answer D).

Chronic obstructive pulmonary disease (COPD) results from an abnormal inflammatory response in the lungs. Which immunosuppressant agent is most likely used to treat an acute exacerbation of COPD? A) Cyclosporine B) Tacrolimus C) CD-25 specific antibody D) Corticosteroids

D - COPD is caused by noxious particles or gases which trigger an abnormal inflammatory response in the lung. According to the passage, steroids are used in a number of clinical settings for their general anti-inflammatory properties. Although the other immunosuppressive agents listed have additional clinical use, there is no mention of this in this passage; the passage focuses on the use of these drugs in transplant rejection (which COPD is not). Thus, the best choice is "corticosteroids" (choice D is better than choices A, B, or C).

Exogenous corticosteroids would have what effect on the hypothalamic-pituitary-adrenal axis? A) Increase in both CRH and ACTH B) Increase in CRH and decrease in ACTH C) Decrease in CRH and increase in ACTH D) Decrease in both CRH and ACTH

D - Exogenous corticosteroids would provide negative feedback to the hypothalamic-pituitary-adrenal axis and thus inhibit both CRH (corticotropin releasing hormone, hypothalamus) and ACTH (adrenocorticotropic hormone, anterior pituitary). As a result of the feedback inhibition, CRH and ACTH would both be decreased (choices A, B, and C are wrong).

Several Salmonella species are facultative anaerobes. Assuming that other external conditions are controlled for, would the expected growth rate of a Salmonella colony be slower in the presence or absence of O2? A) In the presence of O2, because aerobic respiration produces CO2, a byproduct that is lethal to facultative anaerobes. B) In the presence of O2, because the final product of aerobic respiration contains more energy than the final product of fermentation. C) In the absence of O2, because the bacteria will need to produce pyruvate decarboxylase, an enzyme required for entrance to the Krebs cycle D) In the absence of O2, because these conditions result in lower production of ATP, which can fuel binary fission.

D - Facultative anaerobes can produce energy in the presence or absence of O2. In the presence of O2, the bacteria undergo aerobic respiration, which produces approximately 19 times as many ATP molecules per molecule of glucose as does anaerobic respiration. (Note that the fermentation step itself does not yield any ATP; it simply regenerates the NAD+ required to continue glycolysis.) ATP is necessary for many cell processes, including reproduction; thus, conditions that lead to less ATP are expected to lead to a lower growth rate. B: The high-energy product formed under both aerobic and anaerobic conditions is ATP; aerobic conditions simply allow more of it to be produced.

An effective and efficient method for the delivery of an antisense gene could be: A) Orally as an emulsified product. B) Microinjection into individual body cells. C) Intravenously as a nonantigenic, blood-stable product. D) Infection of an embryo by a virus modified to carry the gen

D - For an antisense gene to work, it must be incorporated into the cell in which it will perform its job so that its product is available to hybridize with the sense mRNA that needs to be blocked. Of the options listed, the best way to deliver the antisense gene into all the cells of the individual would be to infect an embryo with a virus that carries the antisense gene. The appropriate virus could become incorporated into the genome of the embryonic cells, thus causing all cells derived from these embryonic cells to contain the antisense gene. Therefore, D is the best answer. A is incorrect because the gene would most likely be destroyed in the acidic environment of the stomach. B is not the best option because microinjection of a gene into all cells would be extremely impractical. C is incorrect because even though the gene could be injected as a blood-stable product, it is not likely that the drug would enter its intended target cells.

Oxidation of ethanol by alcohol dehydrogenase results in a change in the NAD/NADH ratio. This ratio consequently affects the ratio of pyruvate and lactate acid in the process of fermentation. how will the oxidation of alcohol dehydrogenase affect gluconeogenesis? A) Increase gluconeogenesis, because of increased NAD levels B) Increase gluconeogenesis, because of decreased NAD levels C) Decrease gluconeogensis, because of increased NAD levels D) Decrease gluconeogenesis, because of decreased NAD levels

D - If ethanol is oxidized by alcohol dehydrogenase, this implies that NAD is reduced to NADH, as oxidation reactions are always paired with reduction reactions. An increase in NADH, drives the reduction of pyruvate to lactic acid, consequently decreasing the amount of pyruvate. Since pyruvate is the starting material in gluconeogenesis, reduced pyruvate would result in a decrease in gluconeogenesis.

Which of the following changes will increase the rate at which a mammalian organism loses body heat to the environment? A) Vasoconstriction of the organism's superficial blood vessels B) Vasoconstriction of the organism's pulmonary blood vessels C) Replacement of the organism's lean muscle mass with superficial fat D) Increase in the organism's rate of pulmonary ventilation

D - In biological contexts, heat transfer to the environment is facilitated by processes like ventilation (ie, inhalation and exhalation) and the flow of blood through superficial anatomical structures. The regulation of superficial blood flow via vasoconstriction and vasodilation adjusts blood flow to the skin, which is in physical contact with the ambient air. Therefore, changes in skin blood flow modulates conductive heat loss to the environment. Heat transfer to the environment through ventilation occurs through a similar mechanism except that conductive heat transfer to inhaled air is followed by convective heat transfer through exhalation. Therefore, increasing the rate of respiration increases the rate of heat loss to the environment. (Choice A) Vasoconstriction would reduce blood flow to superficial areas and diminish the rate of heat loss to the environment. (Choice B) Vasoconstriction would reduce blood flow to the lungs and diminish the rate of heat loss due to respiration. (Choice C) Replacing lean muscle mass with superficial fat would increase the thickness of the superficial structures that act as thermal insulators, decreasing the rate of heat loss to the environment.

In a culture of mammalian skeletal muscle cells, the consumption of oxygen and glucose is measured. Which of the following would occur in response to inhibition of electron transport? A) Oxygen consumption will increase, and glucose consumption will decrease. B) Oxygen consumption will increase, and glucose consumption will increase. C) Oxygen consumption will decrease, and glucose consumption will decrease. D) Oxygen consumption will decrease, and glucose consumption will increase.

D - In the absence of electron transport, oxygen is not needed (it is the final electron acceptor in the transport chain) so the consumption of oxygen would decrease (choices A and B are wrong). Since the electron transport chain is not available to make energy (ATP) the cells will rely solely on anaerobic respiration - glycolysis. Further, since the energy demands of the cells have not changed, and since glycolysis makes fewer ATP than electron transport, the rate of glycolysis will have to increase to keep the level of ATP normal. Thus, glucose consumption would increase (choice D is correct and C is wrong).

What is the purpose of using SDS as a reagent when running an SDS page protocol? A) It ensures that the largest proteins are the first to reach the bottom of the gel. B) It reduces S-S bonds between cysteine residues. C) It denatures the primary structure of the protein. D) It linearizes the protein provides uniform charge to mass ratio for each sample.

D - SDS is a detergent commonly administered before proteins separation. SDS denatures proteins making it linear. As an anionic substance, it also codes the protein with negative charge. This allows proteins to be separated on the basis of charge allowed, without deviations and charge skewing the results. A: Larger proteins will travel slowly through a polyacrylamide gel, it is the smallest proteins that should reach the end first. B: This would be true of a reducing agent, not SDS. C: SDS denatures secondary, tertiary, and quaternary structure. However, the primary structure is simply the amino acid sequence of the protein. Denaturing this level of structure is both unattainable for SDS and undesirable for the overall procedure.

Which of the following is a true statement about the diaphragm? A) It contains both skeletal and smooth muscle cells. B) Its effector neurotransmitters are norepinephrine and acetylcholine. C) It is innervated by the phrenic nerve and autonomic nervous system. D) It receives neural signals from the cerebral cortex and the brain stem.

D - The diaphragm is purely skeletal muscle (choice A is false) and as such, ACh is the only neurotransmitter used (choice B is false). It's innervated only by the phrenic nerve, not autonomic nerves (choice C is false). The phrenic nerve originates both in the cerebral cortex, for voluntary breathing, and in the brain stem for involuntary control (choice D is correct).

Order the stages of development from fertilization to organogenesis: A) Blastula, zygote, morula, gastrula B) Zygote, blastula, morula, gastrula C) Morula, blastula, gastrula, zygote D) Zygote, morula, blastula, gastrula

D - The diploid cell produced by fertilization is known as a zygote; following multiple divisions with no cell growth, the ball of cells is a morula, further division results in a blastula; finally an invagination into this structure results in 3 tissue layers called the gastrula

Folate is a common supplement given to pregnant women to prevent neural tube defects. The most likely reason for this is: A. Folate is involved in action potential transmission from mother to fetus. B. Folate is directly involved in erythropoiesis. C. Folate is critical for proper development of the mesoderm. D. Folate plays a role in ectodermal induction.

D - The mesoderm gives rise to structures such as bone, cardiac muscle, skeletal muscle, smooth muscle, and tissues involved in the excretory and reproductive systems. Folate is not crucial for this development directly, and the information presented in the question stem does not support this statement. Neural tube defects are defects in the central nervous system. The nervous system is derived from the ectoderm. It can be concluded from the given information that folate is important for neurulation, or the induction of the ectoderm to differentiate into the nervous system. Remember, the ectoderm (shown below) is the outer layer of the gastrula. It gives rise to the nervous system, epidermis, hair, nails, teeth, and sweat glands.

Effective vaccines can provide lifelong immunity to certain pathogens without ever eliciting symptoms. Of the choices below, the molecule from which the most effective vaccine could be developed is: A) A highly variable glycoprotein present on the surface of mycobacterium tuberculosis. B) A segment of viral DNA containing a gene that is conserved across all strains of Epstein-Barr virus. C) Intact botulism toxin, produced by every strain of clostridium botulinum. D) Whole polio virus particles subjected to intense heat.

D - Using a whole viral particle increases the chance of presenting a viable antigenic domain to which antibodies may effectively bind. Intense heat is frequently used to denature viral enzymes and melt viral genetic material, eliminating its pathogenic potential. A- Successful vaccines, according to this question, should provide lifelong immunity. A highly variable domain is unlikely to remain unchanged across all strains of the bacteria during the span of a human life. While a vaccine developed from this molecule may provide immunity for a few years, a new strain of M tuberculosis is likely to display in new in unrecognizable form of the glycoprotein. B-Since DNA is concealed within the viral capsid, it does not represent an ideal target for recognition by immune cells. C-Although antibodies may be able to recognize this toxin and react accordingly such a toxic vaccine is likely to have noticeable negative effects on the patient.

The color of mucus produced by coughing or sneezing is usually dependent on cellular composition. Mucus may be green if it contains high levels of myeloperoxidase, and enzyme responsible for the synthesis of reactive oxygen species. A patient who has been coughing up green sputum likely has high numbers of which cell type in his lungs? A) Helper T lymphocytes. B) Memory B lymphocytes. C) Platelets. D) Neutrophils.

D - neutrophils are the only cell type listed that utilize free radicals to kill pathogens. Thus, they are the only cells likely to contain myeloperoxidase.

Germ Layers

Endoderm (innermost layer): organs that assist in digestion (eg, liver, pancreas) and the epithelium (lining) of the respiratory and digestive tracts Mesoderm (middle layer): musculoskeletal system, circulatory system, and portions of the reproductive and urinary systems Ectoderm (outermost layer): nervous system, integumentary system (skin, hair, nails), and lining of the nostrils, mouth, and anus


Kaugnay na mga set ng pag-aaral

Quiz 10 Questions- 87, 88, 89, 91

View Set

Beginnings: Black and White, Color

View Set

Fractions, Percent's, Decimals, and Ratios

View Set

Topic Two, Lesson Three: Centrally Planned Economy-Economics

View Set